Praxis Chapter Questions

¡Supera tus tareas y exámenes ahora con Quizwiz!

Which of the following is not true with regard to treatment of children with language disorders? A. Because many children with language disorders have difficulties with working memory, clinicians should conduct therapy primarily through the auditory modality and not be concerned with incorporating tactile or visual activities into therapy. B. Collaboration with classroom teachers is important in helping children generalize treatment target behaviors C. it is helpful when appropriate to incorporate reading and writing intervention into language therapy D. A child's chronological age is not always the best predictor of the kind of treatment that will be appropriate; current skills is more reliable indicator

A. Because many children with language disorders have difficulties with working memory, clinicians should conduct therapy primarily through the auditory modality and not be concerned with incorporating tactile or visual activities into therapy.

Using brown's morphemes as a reference, which utterance below represents two morphemes? A. Daddy's B. Cupcake C. Choo-choo D. no no!

A. Daddy's

Xu Fang is a 7-year-old girl in an all-English-speaking second-grade classroom. Xu's parents immigrated from mainland Chini 8 years ago; Xu was born in the United States. She to an all-English kindergarten speaking only Mandarin; kindergarten was her first exposure to English on a regular basis. Xu had no prior preschool experience in either Mandarin or English. The second-grade teacher has referred Xu for a speech-language evaluation because he says that although she interacts well with her English-speaking classmates on the playground, she is "behind" her classmates in written language skills (e.g., spelling, reading). Based on Xu's background, you can state that: A. because BICS develops faster than CALP, Xu is probably developing in an appropriate manner, and at this time, evaluation for a possible language impairment is not necessary B. because Xu has been in an all English speaking classroom setting for at least 2 years, her CALP should be as well developed as her BICS. Her difficulties are red flag, and a speech-language assessment should be conducted C. Xu's CALP should actually develop faster than her BICS, because Chinese parents are known to focus on academic skills at home; thus, her BICS-CALP gap is a red flag that she may have an underlying language impairment D. Xu's BICS-CALP gap may indicate a possible intellectual disability, and she should be evaluated by the school psychologist

A. because BICS develops faster than CALP, Xu is probably developing in an appropriate manner, and at this time, evaluation for a possible language impairment is not necessary

You are working in a school and are told that a new child with an IEP has transferred to your school and will be needing language intervention. On her IEP, it states that she has a "pragmatic language disorder" which one of the following problems are you MOST likely to see in this student? A. difficulties in allowing a conversational partner to have a turn B. problems using complex and compound sentences C. vocabulary deficits D. word retrieval problems

A. difficulties in allowing a conversational partner to have a turn

Lucien N., a 22-year-old male, is hospitalized after sustaining a TBI from a motor vehicle accident. There is no injury to the cerebellum, brainstem or peripheral nerves. When assessing Lucien, the clinician might expect to find: A. dysarthria, confused language (e.g., confabulation), auditory comprehension problems, confrontation naming problems, perseveration of verbal responses, pragmatic language problems, and reading and writing difficulties B. dysarthria, confused language (e.g., confabulation), auditory comprehension problems, no confrontation naming problems, and aggrammatic or telegraphic speech C. confrontation naming problems, perseveration of verbal responses, pragmatic language problems, intact reading and writing skills, and echolalia D. severely impaired fluency, severe echolalia, aggrammatic and telegraphic speech, and intact auditory comprehension skills

A. dysarthria, confused language (e.g., confabulation), auditory comprehension problems, confrontation naming problems, perseveration of verbal responses, pragmatic language problems, and reading and writing difficulties

The mother of danny, a 3 year old who is speaking very little, has been working with an SLP on some language stimulation techniques to build danny's expressive language skills. One day when they are driving, danny points to the skill and says excitedly, "plane sky!" His mother responds "yes i see that big silver plan flying up in the blue sky! wow!" She had just used the technique of A. extension B. parallel talk C. expansion D. recasting

A. extension

Baselines: A. help establish the initial (natural) level of client's responses B. help measure responses that are generalized to natural settings C. may replace probes D. Are not necessary for EBP

A. help establish the initial (natural) level of client's responses

You have a new job in a school district where you are serving preschoolers, elementary students, and teens. You will remember that Part C of the individuals with disabilities education act A. is a federal grant program that assists states in operating a comprehensive statewide program of early intervention services for infants and toddlers with disabilities, ages birth through 2 years, and their families. B. is a focused early diagnostic program whose goal is to identify language delayed preschool children and refer them to head start C. guarantees appropriate augmentative, alternative communication devices for children with CP and other medically based conditions D. mandates IEPS that provide for language-based work-study programs for language impaired student in high school.

A. is a federal grant program that assists states in operating a comprehensive statewide program of early intervention services for infants and toddlers with disabilities, ages birth through 2 years, and their families.

Justin is a 7 year old second grade child who has been diagnosed with Asperger's syndrome. Since he was a toddler, he has had language difficulties. He has just transferred to your school district, and the speech-language services have been recommended for him. Justin's parents are anxious for him to begin therapy as soon as possible. You read over the file of reports written by personnel from his previous school district and meet with your school's student study team to discuss Justin and recommend the best possible program for him in your school district. The report from the previous SLP says, among other things, that Justin's language sample showed that he had difficulty with forms, such as -er and -est. Problems with these forms reflect poor A. morphologic skills B. pragmatic skills C. literacy skills D. semantic skills

A. morphologic skills

You are seeing a 9-year-old boy, Emile, whose PPVT-4 score is one year above age level. Emile appears to be performing appropriately in the classroom. His teacher reports that he is at grade level in most subjects. However, he often interrupts others and irritates his listeners; as a result, he is avoided by many peers. His mother reports that he is not invited to other children's birthday parties and that she has heard that other mothers view him as rude and disrespectful. Treatment should focus on increasing A. pragmatic skills B. syntactic skills C. morphologic skills D. semantic skills

A. pragmatic skills

The range in a distribution can be defined as A. the difference between the highest and lowest scores in a distribution B. the middle 50% of scores of a distribution C. the middle 50% of scores in a distribution divided by 2 D. the variance plus the difference between the highest and lowest scores in a distribution

A. the difference between the highest and lowest scores in a distribution

Jonathan is a 5 year old boy with a language impairment. He has a german shepherd at home named angel. The neighbors have a rottweiler, and his grandma has a poodle. But johnathan only calls angel a dog, he does not use the word dog to refer to his neighbors rottweiler or his grandma's poodle. We can say that jonathan is demonstrating the pattern of A. underextension B. overextension C. restricted semantic categories D. overcategorization

A. underextension

An octave is

An indication of the interval between two frequencies

A child says "red crayon." This is an example of which type of semantic relations?

Attribute + entity

A clinician is asked to give a workshop to graduate students about evaluation of patients with swallowing disorders. She discusses evaluation in depth. Which one of the following facts in the clinician's workshop would be inaccurate? A. An ultrasound examination can measure oral tongue movement and hyoid movement. B. A manometric assessment can assess the preparatory phase of the swallow using posterior and lateral plane examinations. C. An electromyographic assessment can be conducted by attaching electrodes on structures of interest (e.g., oral, laryngeal, or pharyngeal muscles). D. A laryngeal examination can be conducted with indirect laryngoscopy or endoscopic examination to inspect the base of the tongue, vallecula, epiglottis, pyriform sinuses, vocal folds, and ventricular folds.

B. A manometric assessment can assess the preparatory phase of the swallow using posterior and lateral plane examinations.

Which one of the following is not true? A. standardized language tests provide a means of quantifiable comparison of a child's performance to that of groups of children in a similar age category B. Standardized language tests help sample behaviors adequately, providing multiple contexts for sampling target-language behaviors C. with young children, we want to examine play skills (among other things) D. In language sampling, some clinicians calculate a type-token ratio which represents the variety of different words a child uses expressively

B. Standardized language tests help sample behaviors adequately, providing multiple contexts for sampling target-language behaviors

As a clinician in a medically based private practice, you receive a referral of 37-year-old Jan, who has been a physical education teacher for the last 15 years. Jan works part time as a telemarketer, and, according to her husband, she "is glued to her cell phone." Jan also sings in the church choir. She has been hoarse for several years and tells you during the case history that "I've ignored the way I sound—it's just me. I haven't felt like I've needed to change anything." However, Jan shares that lately she has been feeling a lot of pain, and the hoarseness is substantially worse. She says, "Sometimes when I talk, it's almost like there's a 'double voice.'" Jan tells you she is worried because her job depends upon being able to yell and raise her voice on the playground as she teaches physical education. She is worried that she might lose her job if she can no longer yell. You immediately refer Jan to an otolaryngologist for a thorough examination of her vocal folds. You then proceed to do your own perceptual and instrumental evaluation. You arrive at a number of findings, including the fact that Jan has increased laryngeal dysphonia, airway resistance, and a maximum phonation time of 7 seconds. You think that Jan is a probable candidate for phonosurgery, but you will wait for the otolaryngologist's diagnosis and recommendations. When you assess Jan, you indeed find the presence of "double voice." What is the perception of two distinct simultaneous pitches during phonation? A. Diplophonia B. Strain-strangle C. A possible indicator of unilateral vocal fold paralysis D. Cul-de-sac resonance E. A symptom of adductor spasmodic dysphonia

B. Strain-strangle

A clinician is providing services for a school age child, Myron, who uses an AAC device. Myron's parents and teacher report that he is having trouble with the device; other frequently do not understand what he is trying to communicate. He is showing increasing signs of frustration by facilitating his communication with other people. To help Myron communicate more effectively with others so that they understand his messages better, the clinician needs to make sure that the symbols on Myron's AAC device are A. PECS friendly B. Transparent C. non-iconically opaque D. opaque

B. Transparent

Mario is a 4 year old head start preschool child whose teacher has observed some red flags in his language development. You decide to interview Mario's parents and observe him in the classroom setting. You also decide to conduct a language sample. You want to conduct a language sample with Mario because: A. you can use the results to obtain quantitative data such as percentile ranks, standard deviations, as required by the school district to enroll him in speech services B. a language samples primary diagnostic function is to evaluate the child's language in actual daily settings C. the teacher has requested it, and by law, you need to carry out assessments requested by classroom teachers D. language samples are mandated by the common core state standards

B. a language samples primary diagnostic function is to evaluate the child's language in actual daily settings

The SST believes that Justin might profit from SCERTS approach to intervention, which involves A. an emphasis on improving social communication, implementing ongoing evaluation of regulatory behavior, and training syntactic skills for increased communication success B. an emphasis on the importance of targeting goals in social communication and emotional regulation by implementing transactional supports, such as visual supports, environmental arrangements, and communication style adjustments C. an emphasis on improving semantic communicative effectiveness, regulation of emotional state, and transactional environmental supports, such as increase auditory cues D. a holistic approach that encourages social communication, effectiveness in regulating transactions, and successful dialogue with others in the environment.

B. an emphasis on the importance of targeting goals in social communication and emotional regulation by implementing transactional supports, such as visual supports, environmental arrangements, and communication style adjustments

Tonia is a 6 year old with ASD. As you work with her, you find that she frequently says what you just said. What you are observing is Tonia's use of... A. extrapolated utterances B. immediate echolalia C. delayed echolalia D. semantic replication

B. immediate echolalia

You are offering language treatment to a 7-year-old boy, Cameron. Among other things, you are working on the goals of helping Cameron accurately produce the regular plural -s and regular past tense -ed. You need to measure the generalized production of those skills when you withhold treatment for correct responses. The procedure you would use to achieve this is: A. the pre-and posttest results of a standardized test administered at the beginning and conclusion of treatment B. probes C. dynamic assessment D. base rates

B. probes

the slp has noted that danny does speak, he appears to have word retrieval difficulties. The SLP decides to target this in therapy. In targeting word retrieval skills, she is working on the area of A. morphology B. semantics C. pragmatics D. phonology

B. semantics

Mr. Nehru is a gentleman from India who has had a stroke and now has aphasia. His family reports that before the stroke, he spoke both Hindi and English fluently. In planning for therapy, you, as a monolingual English-speaking speech-language pathologist, think about the possibility of incorporating work on English idioms. Which of the following do you need to consider in terms of whether or not to make comprehension and expression of English idioms a treatment goal? A. Mr. Nehru's oral and written abilities in both English and Hindi B. your own interest in English idioms and whether or not you consider them important in therapy for a stroke patient C. whether Mr. Nehru will return to his work setting, where his colleagues speak English, or whether he will now spend all of his time at home, where his family speaks primarily Hindi D. Mr. Nehru's educational and vocational history

B. your own interest in English idioms and whether or not you consider them important in therapy for a stroke patient

You are observing a clinician in a private setting. He specializes in child language disorders and serves elementary-age children from a variety of local public schools. When you observe this clinician doing therapy, you see that he has a well-structured reward system for each child. Some children receive a fruit loop for each correct response they make; others work to earn stickers and even small toys. This clinician has written down each specific behavior that he wishes to elicit from each child, with a percentage of accuracy attached. For example, an objective for one child reads, "when presented with a picture of two or more objects, Jimmy will label the picture using plural -s 80% of the time." This clinician probably subscribes to which theory of child language development?

Behaviorist

You are assessing the expressive language skills of a 4 year old with delayed language. One of the things he says is "my birthday party was fun- we ate cake and cookies" This would count as A. 10 words, 10 morphemes B. 10 words, 12 morphemes C. 10 words, 11 morphemes D. 10 words, 9 morphemes

C. 10 words, 11 morphemes

A third-grade teacher refers 8 year old Allyson to you. The teacher is concerned because " allyson's verbal expression skills just are not what they should be. When she talks, she speaks in real simple sentences. Sometimes, I feel like I'm dealing with a kindergartener, not a third grader." When you speak with allyson's parents they say "allyson never was much of a talker. She talked late-later than her brothers and sisters. But she was always well behaved and we never thought she had a problem." You decide that as part of your assessment of allysons language skills, you will gather and analyze a language sample using TTR. When you eventually calculate TTR based on her language sample, you find that her TTR is 0.31. You conclude that... A. Allyson is normally developing her syntactic skills B. Allyson is delayed in pragmatic skills C. Allyson is low in her lexical skills or the number of words she uses expressively D. Allyson has receptive morphological problems

C. Allyson is low in her lexical skills or the number of words she uses expressively

Because of his diagnosis of Asperger's syndrome, you can assume that Justin will probably have characteristics such as: A. generally below average intelligence, a lack of responsiveness to and awareness of other people, and sterotypical body movements B. a preference for solitude and objects rather than people, a lack of interest in nonverbal and verbal communication, tantrums, head banging, and insistence on routines C. a seemingly excellent vocabulary; seemingly normal syntactic skills; and speech that often seems to be a "monologue" in which justin does not allow his conversational partner to take turns D. an IQ of 70 or below, speech characterized by monologues, and head banging

C. a seemingly excellent vocabulary; seemingly normal syntactic skills; and speech that often seems to be a "monologue" in which justin does not allow his conversational partner to take turns

In a preschool setting, a typically developing child walks up to a child with SLI and says, "let's play house. you be the mommy, i'll be the daddy and we'll make dinner. then the grandma will put the baby to bed" the typically developing child is suggesting that they engage in which type of play A. parallel B. associative C. collaborative D. side by side symmetrical

C. collaborative

A monolingual English-speaking speech-language pathologist is working in a Head Start in a city with many CLD children. In the last year, CLD children from over 15 different cultural and linguistic groups have come to the Head Start speaking only their primary language. The ideal plan for dealing with the needs of these children is to: A. encourage them not to speak their primary languages and to speak English as much as quickly as possible B. tolerate the children's use of their primary languages and speak to them solely in English, hopefully that they will eventually "pick up" English and discontinue using their primary languages C. hire bilingual aides from the neighboring communities and use their services to help the children maintain their primary languages and learn English also D. assume that these children probably have language-learning disabilities in their primary languages and hire bilingual SLPS to assist in remediation

C. hire bilingual aides from the neighboring communities and use their services to help the children maintain their primary languages and learn English also

You are seeing a 6 year old child, Tyler, with specific language impairment. When you assess Tyler, you find that he has adequate language comprehension. He is able to follow directions, understand vocabulary, and comprehend sentences of appropriate length and complexity for his age. However, his teacher and parents report that he has "no friends" and that they are concerned about his social skills. When you observe Tyler several times on the school playground, in the classroom, and in the school cafeteria, you see that, while he is well behavioral and non-disruptive, he does not initiate interactions with others. Treatment should focus on A. increasing MLU B. working on bound morphemes C. increasing assertiveness in conversation D. increasing sentence complexity

C. increasing assertiveness in conversation

You are working with an adolescent, Alyssa, who has receptive and expressive language problems. She is getting Ds in most of her classes at the junior high school and has few friends. In therapy, it would be best to target A. increasing auditory memory skills B. increasing the use of complex sentences containing subordinate clauses C. increasing social use of language and collaborating with the classroom teachers D. increasing the understanding and use of figurative language

C. increasing social use of language and collaborating with the classroom teachers

Parents bring their son Derek to you. He is 30 months old and only says a few words. The pediatrician has told them to not worry and just give Derek time, saying "he is a boy, after all, and they usually develop language more slowly." But derek's parents are still concerned. The best recommendation you could give them would be: A. do nothing and follow the pediatricians recommendation to give him more time to develop B. give them a home language stimulation program, and tell them to come back in a year for a re-evaluation C. recommend an immediate full evaluation of language D. send derek and his parents to a psychologist for evaluation of his cognitive skills

C. recommend an immediate full evaluation of language

Consuelo is a Mexican American Spanish-speaking 6-year-old girl who is in the process of learning English. Her parents immigrated from Mexico three years ago; thus, Consuelo was exposed first to Spanish at home and was exposed to English in kindergarten at the age of 5 years 3 months of age. Consuelo attended an English-speaking preschool. The classroom teacher shares that she thinks Consuelo may have an articulation disorder, but the teacher is not sure. The teacher provides you with some examples of things that Consuelo has said in the past two to three weeks. As you look at these examples, which one of the following would *NOT* be typical for her in terms of predictable productions based on Spanish influence? A. t/th substitutions in word-initial positions (e.g., tin/thin) B. devoicing of final consonants (e.g., beece/bees) C. v/f substitution in word-initial and word-final positions (e.g., vine/fine; roove/roof) D. y/dʒ substitution (e.g. yava/java)

C. v/f substitution in word-initial and word-final positions (e.g., vine/fine; roove/roof)

The cranial nerve that innervates the larynx and also innervates the levator veli palatini, palatoglossus, and palatopharyngeus muscles is

CN X

You obtain 50 utterances from Mario. Your analysis of the language sample shows that Mario used 100 words and 120 morphemes. Thus, his average MLU is A. 5.0 B. 4.5 C. 2.0 D. 2.4

D. 2.4

A clinician is a member of a cleft palate and craniofacial team that asks her to conduct an objective assessment of a 6-year-old child's velopharyngeal mechanism. The clinician decides to do nasopharyngoscopy, in which the nasopharyngoscope is passed through the middle meatus and back to the area of velopharyngeal closure. What will this procedure enable the clinician to observe? A. adenoid pads and anterior pharyngeal walls as the child prolongs /s/ B. The child's posterior and lateral pharyngeal walls, as well as the adenoid pad as the child sustains /a/ C. The child's nasal aspect of the velum and the adenoid pad as the child produces CVC words D. The child's posterior and lateral pharyngeal walls, as well as the nasal aspect of the velum and the adenoid pad as the child produces sentences

D. The child's posterior and lateral pharyngeal walls, as well as the nasal aspect of the velum and the adenoid pad as the child produces sentences

A researcher teaches a new book reading program to caregivers of children on the autism spectrum and evaluates the children's literacy skills one year later. The researcher's goal is to evaluate whether or not there is a relationship between caregivers' implementation of the program and children's literacy skills. The researcher finds that there is an r =.15 correlational relationship between caregivers' reported implementation of the program and children's literacy skills. The researcher can safely conclude that: A. There is a strong positive correlation between the caregivers' implementation of the program and children's language skills. B. There is a strong negative correlation between the caregivers' implementation of the program and children's language skills. C. There is a moderately significant cause-effect relationship between the caregivers' implementation of the program and children's language skills D. There is no significant relationship between the caregivers' implementation of the program and children's language skills.

D. There is no significant relationship between the caregivers' implementation of the program and children's language skills.

A child who shows slow, writhing, involuntary movements has which type of cerebral palsy? A. spastic B. Mixed C. ataxic D. athetoid

D. athetoid

A clinician was working with a 4-year-old boy, Emile, who stuttered. During treatment, the clinician found that every time she asked Emile to imitate a long phrase or sentence, he got up out of his chair and tried to wander around the room he would not look at stimulus items. He would complain and say, "That's too hard!" Within two sessions, Emile's uncooperative behaviors increased and he seemed to be getting worse. To control Emile's undesirable behaviors, the clinician simplified the target skill; she modeled simpler, shorter, sentences with the goal of increasing the utterance length more gradually and slowly than she had done previously. Consequently, Emile's undesirable behaviors decreased and were eventually eliminated. This show's that Emile's undesirable behaviors were: A. being effectively punished B. being positively yet indirectly reinforced during treatment sessions C. being reinforced on intermittent, variable ratio schedule D. being negatively reinforced in treatment sessions

D. being negatively reinforced in treatment sessions

A school based SLP is conducting classroom based intervention with several students diagnosed with specific language impairment. This means that the SLP is A. conducting pull out intervention, in which she brings the students to the therapy room and uses classroom materials as part of a small group session B. conducting pull out intervention in which she brings a student to her therapy room and uses classroom materials as part of an individual session C. teaching language skills to the whole class; the students with SLI are part of this class D. going into the classroom and helping language impaired students, individually or in a small group format to achieve classroom curriculum goals

D. going into the classroom and helping language impaired students, individually or in a small group format to achieve classroom curriculum goals

A school-based clinician is assessing the velopharyngeal adequacy of Janie K, a 17-year-old immigrant high school student from the Phillipines. Janie was born with a cleft of the palate and lip; there was no repair until Janie's family came to the United States when she was 16 years old. In the Philippines, Janie and her family lived in a rural area where surgery was unavailable. Though the repair surgery in the United States a year ago was successful and Janie now has a more aesthetically pleasing appearance and better speech, there is still audible nasal emission and hypernasality when she speaks. The clinician plans to refer Janie to a local craniofacial team, but she still wants to conduct as thorough an examination as she can. Despite the lack of instrumentation available at her school site, the clinician does have access to an oral manometer. She uses this to provide a beginning point from which to refer Janie to the craniofacial team. After obtaining a ratio by comparing pressures achieved in the nostrils-occluded and the nostrils-open conditions, the clinician concludes that Janie especially needs to be referred to the craniofacial team for possible further surgery or a pharyngeal flap. When she did oral manometry, the clinician probably found that Janie A. had a ratio of 1.0 B. had a ratio of 1.2 C. had a ratio of .96 D. had a ratio of .87

D. had a ratio of .87

You move to a new elementary school and begin seeing the children on the caseload at the school. One child, who is being treated to "increase semantic skills," has four goals listed on her IEP. Which one of these goals is inappropriate? A. increase types and numbers of words the child uses in the classroom B. increase specific word usage and decrease usage of nonspecific words, such as this, that, thing C. decrease overextensions of words D. increase use of appropriate discourse skills, turn taking, and conversational repair strategies

D. increase use of appropriate discourse skills, turn taking, and conversational repair strategies

A child from Apple City transfers to Middleton City, and his file indicates that he has been receiving speech-language services in Apple City. To the chagrin of the Middleton City clinician, some pages of the report from the SLP in Apple City are missing. However, on the first page, it is indicated that this child has Moebius syndrome. He also has a history of frequent hospitalizations. The Middleton City clinician can probably expect to find that this child: A. has low muscle tone, a history of early feeding difficulties, initial failure to thrive, obesity after the first year, and underdeveloped genitals B. has underdeveloped facial bones, including mandibular hypoplasia, malar hypoplasia, dental malocclusion, and downwardly slanted palpebral fissures C. has a small maxillary structure, sphenoethmoidal synchondroses, ocular hypertelorism, facial asymmetry including a tall forehead and brachycephaly D. may have delayed language and an articulation disorder, as well as bilabial paresis and weak tongue control for lateralization, elevation, depression, and protrusion; a mask-like face; a history of feeding problems in infancy; and unilateral or bilateral paralysis of the abductors of the eye

D. may have delayed language and an articulation disorder, as well as bilabial paresis and weak tongue control for lateralization, elevation, depression, and protrusion; a mask-like face; a history of feeding problems in infancy; and unilateral or bilateral paralysis of the abductors of the eye

Jeannette is a 4 year old girl with specific language impairment. At preschool the teacher says "before you put on your jacket to go outside, be sure to get your snack." Jeannette puts her jacket on first and then tries to get her snack. The teacher becomes angry and believes jeannette is not following directions. Jeannette is showing difficulty with A. phonological awareness B. pragmatics C. syntax D. order of mention

D. order of mention

A clinician is working with parents on home language stimulation activities for their 3 year old child, whom is language delayed. The child needs to increase her expressive language. Her MLU = 2 yr. Recommend that the parents use a technique in which they describe and comment on what the child is doing. For example, "You are making the car go fast" or "That pig is pink". This technique describes: A. self talk B. expansion C. expatiation D. parallel talk E. joint reference

D. parallel talk

When a speaker is producing a vowel and the vowel is being acoustically analyzed, one can state as a general rule that:

F1 varies mostly as a result of tongue height and F2 varies mostly as a result of tongue advancement (variation in the anterior-to-posterior position of the tongue in the oral cavity)

You have been asked to give a workshop to a group of parents of infants who attend a developmental nursery. The parents are interested in what they can do to communicate more successfully with their infants. Most of the infants are between 1 and 10 months of age. Most of the parents do not have much money or access to toys and objects, but you are told that they do spend plenty of time with their babies. You are asked to speak about what specifically these parents can do to successfully interact with their infants in daily routines, such as bathing, dressing, and eating. You will tell these parents...

Ideally, speak to the baby in utterances (child-directed speech) that are higher pitched and have greater pitch fluctuations than ordinary speech

A 7-year-old girl, Ashton, is referred to you by her second-grade classroom teacher, Mr. Alvarez. Mr. Alvarez says that Ashton "doesn't always get along with her peers" and "doesn't know how to hold a decent conversation." You assess Ashton personally and also observe her on the playground during recess and in the cafeteria at lunch time. You see that Mr. Alvarez is right. Ashton has difficulty in conversational exchanges with her peers, and they frequently ignore her. You notice that when talking to you, she seems uncomfortable and doesn't say much, even when you use a variety of interesting games and toys. In therapy, your first priority with Ashton will be to... a. teach her the appropriate use of compound and complex sentences in appropriate contexts b. teach her the appropriate use of allomorphs when presented with pictures of different people and activities c. increase her skills in quick incidental learning so that she can expand her vocabulary d. increase her skills in discourse

Increase her skills in discourse

You are asked to work with a 3 1/2 year old child whose language has been somewhat slow to develop. He is the youngest of 4 children and his parents tell you his older siblings often talk for him. After assessing his language, you find he consistently uses the following morphemes: -ing, in and on, and regular plural -s. Matthew's parents would like to enroll him for therapy because they want him to go to a local preschool and sound like the other kids and have good grammar. Which of the following morphemes would you begin with when he starts therapy?

Irregular past tense verbs

A child using recurrence would say which of the following? a. face dirty b. all gone juice c. more cookie d. doll mine

More cookie

You are conducting an assessment with an incoming kindergartener, Jason, who has difficulty with word endings. Specifically, he tends to omit endings like -est (saying "sad" instead of "saddest"), -ily (saying "angry" instead of "angrily"), etc. He is having difficulty with which specific aspect of language?

Morphology

Which one of the following Piagetian stages, which include object permanence, corresponds with the emergence of a typically developing child's first word? a. preoperational b. formal operations c. sensorimotor d. concrete operations

Sensorimotor

An example of a sentence using an embedded form would be:

The boy who got a haircut looks nice

You are asked to assess Tina, who has Down syndrome. She is 4 years and 10 months old, and her parents tell you that they wish for her to begin kindergarten in the fall (it is July, and school begins in September). You assess Tina's receptive and expressive language skills and find that she has an average MLU of 3.0 and an expressive vocabulary of 350 words. She sustains a topic of conversation about 20% of the time and over regularizes past-tense inflections. You will tell Tina's parents that...

Tina's language skills are generally commensurate with those of a 2 to 3 year old child and starting kindergarten in the fall would probably be difficult for her

A mother comes to you, concerned because her son Jake was born prematurely and had to spend the first few month of his life in a neonatal intensive care unit. Now Jake is 9 months old, and his mother wants to make sure that his language development is "on target for his age." You go to Jake's home to observe him, and you also ask his mother to give you a detailed description of his communication patterns. As you evaluate Jake's language development, you need to remember that one of the following does not occur between 8 and 10 months of age in the typically developing child. a. comprehension of "no" b. using the phrase "all gone" to express emerging negation c. using variegated babbling d. uncovering a hidden toy

Using the phrase "all gone" to express emerging negation

In an adolescent speaker of AAE, which of the following utterances would be an example of the use of the perfective construction, with been used to indicate an action that took place in the distant past? a. "I been had a marble collection when I was 7" b. "our family been gonna do it" c. "I might been coulda done it" d. "He been done it again"

a. "I been had a marble collection when I was 7"

A 74 year old bilingual Asian gentleman has had a stroke, and you are seeing him for therapy. He is recovering both his primary language and his english skills, but you are working only on english. There are no interpreters available, unfortunately, and the family has indicated that they would prefer treatment to be conducted in english anyway, since many of the patient's grandchildren speak english fluently. Which of the following productions would be an example, on the patient's part, of English influenced by his primary language and not the stroke? a. "They coming over here now" b. "I done got to eat breakfast now" c. "She not have no dollar in her purse" d. "We be havin' many fun"

a. "They coming over here now"

Popular forms of amplification today include hearing aids and cochlear implants. Which one of the following is not true about these devices? a. Hearing aids deliver amplified sounds to the ear canal, while cochlear implants deliver electrical impulses (converted from sound) directly to the middle ear b. Digital hearing aids provide a better signal-to-noise ratio than analog aids do. c. Cochlear implants can help prelingual children to make substantial progress through maximizing their potential. d. A consideration in fitting clients with hearing aids is whether they are motivated to use and properly care for the aids.

a. Hearing aids deliver amplified sounds to the ear canal, while cochlear implants deliver electrical impulses (converted from sound) directly to the middle ear

A fifth-grade teacher refers Mia to you. Mia speaks Tongalese. She and her family have been living in the US for 3 years and Mia has been enrolled in US schools for that whole time. However, she has been sick a great deal and missed many days of school. The family is most helpful, and Mia's parents do their best to do assignments with her at home. However, their conversational English is limited, and they do not read or write in English at all. When you talk to the teacher, he states that Mia has friends and gets along well in the classroom, but she especially struggles in the area of reading. The school team meets and decides to utilize RTI approach to discern whether Mia has a language and experiential difference or a language impairment. In the RTI model, a. Mia's teacher will implement the use of scientifically based instruction in the regular education setting to provide her with additional reading support; if this is insufficient to improve her performance, the special education team will evaluate her for possible special education services b. Mia will automatically undergo an extensive special education evaluation that will determine her possible need for pull-out speech-language therapy and academic support from the resource specialist c. Mia will automatically undergo an extensive special education evaluation that will determine her possible need for placement in a self-contained special education classroom d. the classroom teacher will continue "business as usual" providing no extra or additional instruction and assessing whether Mia makes progress

a. Mia's teacher will implement the use of scientifically based instruction in the regular education setting to provide her with additional reading support; if this is insufficient to improve her performance, the special education team will evaluate her for possible special education services

When the speech-language pathologist provides treatment for Rachel to her her become more intelligible, which techniques would be ideal for her? a. Reducing Rachel's rate of speech and increasing her awareness of her speech through audiotapes or videotapes b. Reducing Rachel's rate of speech but not increasing her awareness through audio or videotapes because this could create self consciousness, which could make cluttering worse c. Helping Rachel maintain a rapid rate of speech but working on increasing her intelligibility through emphasizing the final consonants of words d. probing to see if Rachel has negative emotions about her speech and spending most of therapy time helping her deal with these emotions

a. Reducing Rachel's rate of speech and increasing her awareness of her speech through audiotapes or videotapes

Parents in a local school district have asked a clinician to give an in-service on the IDEA. The parents are interesting in the content of the IDEA regarding their parental rights, and so forth. Which one of the following would not be accurate for the clinician to tell parents at the inservice? a. The IDEA discourages serving children with disabilities in general education classroom settings, rather, it encourages school districts to create more specialized pull out programs for such children to best serve their needs b. The IDEA promotes increased, meaningful parental involvement in evaluations, including access to reports and test instruments c. The IDEA mandates the development of alternative assessments for children who cannot participate in standard assessments d. One goal of the IDEA is to promote increased participation of special educators in general classroom settings, including involvement with curriculum

a. The IDEA discourages serving children with disabilities in general education classroom settings, rather, it encourages school districts to create more specialized pull out programs for such children to best serve their needs

Which of the following is not true regarding ESSA? a. Under ESSA, the federal government creates short and long term goals for academic proficiency b. under ESSA, the individual states create short and long term goals for academic proficiency c. ESSA was signed by president obama d. during the ELL second year in the US school, his/her score counts towards the schools proficiency rating

a. Under ESSA, the federal government creates short and long term goals for academic proficiency

An investigator is interested in evaluating the relative effects of three treatments on the production of grammatical morphemes in children with autism spectrum. The investigator also wishes to see if each treatment is effective in its own right. The most appropriate experimental design for this study is a. a group design with four groups b. a pretest-posttest control group design c. multiple baseline design across subjects d. an ABAB design

a. a group design with four groups

You are providing support for a man with Marfan syndrome; he has difficulties with respiration, and you are working on breathing techniques. He shares with you that he and his wife want to have a child, and they are concerned about their child possibly having Marfan syndrome. You refer him to a genetic counselor, who will share with him that Marfan syndrome is a. an autosomal, dominant inherited disorder caused by mutations in the FBN1 gene b. an autosomal recessive disorder caused by mutations in the FBN1 gene c. a parietal deletion syndrome caused by mutations in the FBN2 gene d. not genetically transferred to offspring, so the parents do not need to be concerned

a. an autosomal, dominant inherited disorder caused by mutations in the FBN1 gene

Various objects, pictures, instructions, modeling, prompts, and other stimuli the clinician uses to evoke target responses from clients are called _______. a. antecedents or treatment stimuli b. reinforcing stimuli c. procedural probes d. baselines

a. antecedents or treatment stimuli

A child comes to a clinic with her mother for articulation therapy. The mother tells the clinician that her daughter has Hurler's syndrome. Hurler's syndrome is caused by.. a. autosomal recessive deficiency of X-L iduronidase b. a spontaneous autosomal dominant mutation of FGR2 at 10q25-26 c. autosomal dominant inheritance and deletion in the region of the long arm of chromosome 15 d. an expanded number of cytosine-guanine-guanine nucleic acid repeats on a specific gene on one of the distal ends of the Y chromosome

a. autosomal recessive deficiency of X-L iduronidase

The theory that stuttering is caused by lack of a unilateral hemispheric control of language is the a. cerebral dominance theory b. approach-avoidance theory c. diagnosogenic theory d. hemispheric domination theory

a. cerebral dominance theory

Functional communication assessment targets a. communication in natural or everyday situations b. grammatically correct and complex communication c. comprehension of both daily and academic vocabulary necessary for effective functioning in the "real world" d. phonemically correct communication

a. communication in natural or everyday situations

The highest level of evidence supporting a treatment procedure comes from a. controlled research, systematically replicated b. controlled research, directly replicated c. an experimental study with a large number of participants d. a multiple baseline design study

a. controlled research, systematically replicated

People who stutter and their families a. do not have unique personality traits that explain stuttering b. are diagnosed with clinicially significant anxiety disorders c. have unique temperament related to stuttering d. have obsessive compulsive traits

a. do not have unique personality traits that explain stuttering

Among the traditional and alternative assessment approaches, the one that requires the clinician to offer some treatment before making a full dx and test again to make a final decision is known as: a. dynamic assessment b. authentic assessment c. criterion-referenced and client-specific approach d. standardized-test-based assessment

a. dynamic assessment

A teacher has referred a third grade boy to you for a speech-language assessment. She is concerned because she feels that he is academically "behind his peers." He and his family are cambodian refugees, and they have been in the US for 8 months. Because the boy has been in refugee camps most of his life, his schooling in cambodia was quite limited. His parents tell you that they estimate that he has had approx. 1.5 years of schooling. The teacher is concerned that the boy may have an underlying language impairment, and she wonders if he is eligible for speech-language services. What would be the best combination of assessment techniques to use with him? a. dynamic assessment, language samples in cambodian, and observaitons of his interaction with family members and other cambodian children b. use of the PPVT and Test of language development-primary, translated into cambodian; dynamic assessment; and language samples in cambodian c. use of district developed test for cambodian students in your geographic area and administration of questionnaires to the boy's teachers and family d. use of formal, standardized tests in English combined with observations of the boy's interactions, in cambodian, with peers and family members

a. dynamic assessment, language samples in cambodian, and observaitons of his interaction with family members and other cambodian children

You are serving a child with repaired cleft palate. Marissa is 11 and still somewhat unintelligible, manifesting difficulty with some speech sounds. To help her, you employ a specialized procedure. In this procedure, an orthodontist will design an artificial palate containing 62 embedded electrodes connected to a computer. This will be fitted into Marissa's mouth, and when her tongue contacts the electrods during speech production, articulatory patterns can be seen on the computer screen. This procedure is called a. electropalatography b. electroglottography c. palatomyography d. myomanometry

a. electropalatography

A 3 year old child is receiving therapy for remediation of several phonological patterns. She very frequently says things like tar/kar, do/go, and ti/ki. These productions show that she is using the phonological pattern of a. fronting b. glottal replacement c. stopping d. prevocalic voicing

a. fronting

In assessing the language skills of Rica, a 8 year old speaker of Cantonese and English, the clinician, with the help of a Cantonese interpreter, obtained extensive samples of both the languages. After consulting with Rica's teacher and interviewing her parents, the clinician selected additional vocabulary items and common expressions to be included in systematic assessment. This type of assessment is known as a. functional and criterion-referenced assessment b. norm-referenced and criterion-referenced assessment c. standardized assessment without being norm-referenced d. functional and standardized assessment without being norm-referenced

a. functional and criterion-referenced assessment

Which one of the following would NOT be a feature of this norm-referenced, standardized test? a. generation of information that can be used to create treatment goals and assess treatment progress b. the comparison of a client's score to that of a normative sample c. ensuring of consistency of administration and scoring across examiners d. the provision of systematic procedures for administration and scoring of the test

a. generation of information that can be used to create treatment goals and assess treatment progress

Activities that may be carried out by a trained SLPA include a. implementing treatment plans developed by the supervising SLP, checking and maintaining equipment, and collecting and documenting data for quality improvement b. demonstrating swallowing strategies or precautions, collecting and documenting data for quality improvement, and assisting with speech language and hearing screenings c. writing and modifying treatment plans, assisting with clerical duties and in-service training, and checking and maintaining equipment d. signing treatment plans or assessment reports when the SLP is not available, assisting with speech language and hearing screenings, and assisting with clerical duties

a. implementing treatment plans developed by the supervising SLP, checking and maintaining equipment, and collecting and documenting data for quality improvement

Public Law 99-457 a. increased federal support for services to disabled children 3-6 years of age and provided funding for infants and toddlers b. was enacted in 1986 c. restricts special education services to children with documented disabilities d. requires states to report the number of preschoolers served under different categories of disabilities

a. increased federal support for services to disabled children 3-6 years of age and provided funding for infants and toddlers

You are assessing a 20 year old man who was involved in a MVA with severe head injury. The medical report says that the patient has nonpenetrating head injury. In this case, you expect to observe a. intact meninges b. torn meninges c. no skull fracture d. an open wound

a. intact meninges

Apraxia of speech is often associated with a. lesions in Broca's area b. lesions in Wernicke's area c. lesions in subcortical structures d. lesions in the occipital area

a. lesions in Broca's area

Which of the following is considered an exercise for patients with dysphagia and should not be performed with food? a. masako maneuver b. supraglottic swallow c. mendelson maneuver d. effortful swallow

a. masako maneuver

The articulation therapy approach that emphasizes the syllable as the basic unit of speech production and heavily uses the concept of phonetic environment is a. mcdonald's sensorimotor approach b. the maximal contrast approach c. the metaphon approach d. van riper's traditional approach

a. mcdonald's sensorimotor approach

In oller's stages of infant phonological development, reduplicated babbling precedes a. nonreduplicated or variegated babbling b. expansion c. cooing d. phonation

a. nonreduplicated or variegated babbling

Parents generally report that the onset of stuttering in their children is associated with a. nothing unusual b. stressful family situations c. accidental head injury d. severe illness

a. nothing unusual

Sensorineural hearing loss can be caused by many things. ______, a hearing impairment in older people, results in a sloping, high frequency loss. ____, which also causes sensorineural hearing loss, is accompanied by vertigo and tinnitus. a. presbycusis, meniere's disease b. meniere's disease, otosclerosis c. presbycusis, otosclerosis d. central auditory processing disorder, presbycusis

a. presbycusis, meniere's disease

As a clinician in a medically based private practice, you receive a referral of 37-year-old Jan, who has been a physical education teacher for the last 15 years. Jan works part time as a telemarketer, and, according to her husband, she "is glued to her cell phone." Jan also sings in the church choir. She has been hoarse for several years and tells you during the case history that "I've ignored the way I sound—it's just me. I haven't felt like I've needed to change anything." However, Jan shares that lately she has been feeling a lot of pain, and the hoarseness is substantially worse. She says, "Sometimes when I talk, it's almost like there's a 'double voice.'" Jan tells you she is worried because her job depends upon being able to yell and raise her voice on the playground as she teaches physical education. She is worried that she might lose her job if she can no longer yell. You immediately refer Jan to an otolaryngologist for a thorough examination of her vocal folds. You then proceed to do your own perceptual and instrumental evaluation. You arrive at a number of findings, including the fact that Jan has increased laryngeal dysphonia, airway resistance, and a maximum phonation time of 7 seconds. You think that Jan is a probable candidate for phonosurgery, but you will wait for the otolaryngologist's diagnosis and recommendations. As part of counseling Jan, you share with her that phonosurgery will be necessary. You emphasize that if she continues to abuse her voice, she may have to have repeated surgeries. These repeated surgeries can cause her voice to sound gravelly and rough, which is primarily a result of a. reduced mucosal wave action b. insufficient air supply for optimal phonation c. resonance problems d. papillomas

a. reduced mucosal wave action

A 65 year old man with presbycusis comes to you complaining that when he is in social situations such as parties, people don't speak loudly enough. He says that the noise creates a problem for him in hearing what people are saying. The client has difficulty with a. signal to noise ratio b. auditory discrimination c. figure ground discrimination d. pragmatic skills

a. signal to noise ratio

the muscle that exerts the pull that allows the eustachian tube to open during yawning and swallowing is the: a. tensor veli palatini b. levator veli palatini c. tensor tympani d. stapedius muscle

a. tensor veli palatini

The law stating that employers must provide special equipment for workers with disabilities is a. the americans with disabilities act b. HIPPA c. P.L. 94-142 d. No child left behind

a. the americans with disabilities act

A researcher investigates the effect of rate of speech upon stuttering during sibling interaction. She gathers conversational samples from children who stutter and their siblings. She asks the siblings in the control group tp speak as they normally would at home. Siblings in the experimental group are asked to speak much more quickly than they would at home. The investigator wishes to answer the question whether increased rate of sibling speech causes children to stutter more. In this study, the dependent variable is a. the amount of stuttering done by the children who stutter before and after the siblings increase their rate of speech b. the rate of speech of the siblings in the experimental group c. the rate of speech of the siblings in the control group d. combined amount of stuttering done by the children in both the experimental and the control groups

a. the amount of stuttering done by the children who stutter before and after the siblings increase their rate of speech

A test's ecological validity refers to the extent to which it reflects a. the child's actual, daily environment and life experience b. culture fair assessment practices c. grade-level appropriate content d. procedures which have been proven nondiscriminatory for ELs

a. the child's actual, daily environment and life experience

The cover-body theory of phonation states that: a. the epithelium and superficial, intermediate, and deep layers of the lamina propria vibrate as a "cover" on a relatively stationary "body" This body is composed of the remainder of the TA muscle b. the epithelium, the deep layer of the lamina propria, and much of the superficial layer of the lamina propria vibrate as a "cover" on a relatively stationary "body" which is made up of the remainder of the superficial layer, the deep layer, and the TA muscle c. the superficial layer of the lamina propria and much of the intermediate layer of the lamina propria vibrate as a "cover" on a relatively stationary "body" which is made up of the remainder of the intermediate layer, the deep layer, and the TA muscle d. the epithelium and much of the intermediate layer of the lamina propria vibrate as a "cover" on a relatively stationary "body" which is made up of the remainder of the intermediate layer and the TA muscle

a. the epithelium and superficial, intermediate, and deep layers of the lamina propria vibrate as a "cover" on a relatively stationary "body" This body is composed of the remainder of the TA muscle

A speech-language pathologist who has been working on improving speech intelligibility of non-native speakers of English wanted to collect objective evidence that his clients speech intelligibility was improving. She recruits four speech-language pathology graduate students at the local university to watch "before and after" videos of his accent clients and independently rate each client's speech intelligbility. He finds that for the same client, the intelligibility ratings varied from a 30% to 60% across the four students. In this situation, one can say that, a. there is low interjudge reliability b. there is high interjudge reliability c. there is low interjudge reliability d. there is high interjudge reliability

a. there is low interjudge reliability

In treating older students and adults who stutter, a clinician decides to use the direct stuttering treatment (reduction) procedure; specifically, the clinician then selects a. time-out b. the fluency shaping approach c. approach-avoidance reduction treatment d. the fluent stuttering approach

a. time-out

Sometimes specialists asses the lung volume of voice patients because breath support is inadequate. Specialists can measure ____ of the total volume of air in the lungs; other measurements can include ___ or the amount of air inhaled and exhaled during a normal breathing cycle and ____ or the volume of air that the patient can exhale after a maximal exhalation a. total lung capacity, tidal volume, vital capacity b. vital capacity, tidal capacity, total lung volume c. vital capacity, total lung capacity, tidal volume d. tidal volume, total lung capacity, vital volume

a. total lung capacity, tidal volume, vital capacity

For which of the following disorders disorders of swallowing would it be inappropriate to recommend tilting the head to the strong side? a. unilateral weak pharyngeal constriction b. decreased laryngeal elevation c. decreased upper esophageal sphincter d. incomplete epiglottic inversion

a. unilateral weak pharyngeal constriction

Amanda is a 7 year old with childhood apraxia of speech (CAS). She is frustrated in school because it is hard for her to be understood in class and on the playground, you estimate that she is approximately 60% intelligible. You have just finished graduate school, and Amanda is your first client with CAS. When conducting therapy with her, it will be important to remember to a. use multimodal cueing and tasks that move up in a hierarchical manner b. focus primarily on phonological awareness and auditory discrimination skills c. teach her sign language to supplement speech d. focus most of intervention on listening skills

a. use multimodal cueing and tasks that move up in a hierarchical manner

The surgical method of cleft palate repair that involves two bipedicled flaps of mucoperiosteum, bringing them together, and attaching them to close the cleft is called the a. von langenbeck surgical method b. v-y retroposition c. veau-wardill kilner method d. pharyngeal flap procedure

a. von langenbeck surgical method

A difficulty with cross-sectional studies is that a. what is observed for a particular age group in the study may not hold good for others of the same age who were not in the study b. observations are made of differences within participant groups of different ages to generalize about developmental changes that would occur between participants as they mature c. the same participants are studied over time, and tis is expensive, time consuming, and difficult because participants might drop out of the study d. the total age span of children to be studied is divided into several overlapping age spans, and it is difficult to follow participants from the lower to the upper end of each age span

a. what is observed for a particular age group in the study may not hold good for others of the same age who were not in the study

A clinician in a private practice is approached by the parents of Tommy D., a 5 year old boy. The parents want to place Tommy in kindergarten in the fall but say, "we know there's something wrong with him- we're just not sure what." According to Tommy's parents, he is a "sweet, lovable boy who will go to anybody. He likes to sing a lot, too." Because the parents live in a rural area, health care access has been limited. After seeing Tommy for the first time, the clinician refers his parents to a neurologist because she suspects that Tommy has a syndrome. He is small for his age and has an elfin-like appearance characterized by a small chin, turned up nose, puffiness around the eyes, a long upper lip, and a wide mouth. His teeth are small and widely spaced. The clinician suspects that Tommy has a. william's syndrome b. apert syndrome c. moebius syndrome d. turner syndrome

a. william's syndrome

These are composed of a ring of connective tissue and muscle extending from the tips of the arytenoid cartilages to the epiglottis. They separate the laryngeal vestibule from the pharynx and help preserve the airway. a. ventricular folds b. True vocal folds c. Lamina propria d. aryepiglottic folds

aryepiglottic folds

A clinician administers the language achievement test (LAT) to Bret, who is described as "low" in oral and written language skills. On LAT, Bret's overall score falls in the 25th percentile rank. This means that.. a. out of 100 children, 75 did better than Bret on the LAT and 25 children did worse b. 75% of the children in the LAT's normative sample did better than Bret c. of children in the LAT's normative sample performed about the same as Bret did, but they scored 2-3 points higher on the test overall d. Bret scored about the same as 75% of fifth graders in the LAT's normative sample

b. 75% of the children in the LAT's normative sample did better than Bret

Which statement is true about ASHA's special interest groups? a. Membership in SIGS is free to all ASHA members b. Clinical Specialty Recognition is available only in a few specialty areas c. A clinical specialty recognition is essential to assess and treat clients with a specialized disorder d. each SIG is concerned with a specific disorder of communication

b. Clinical Specialty Recognition is available only in a few specialty areas

You are working in a school district where there is an influx of immigrants from Russia. These immigrants have come to your area because there are Russian churches that have sponsored them. In the last few years, over 50,000 Russian families have moved in. Teachers are beginning to refer some Russian students to you for assessment for language impairment, saying that the students are "not learning English fast enough." You consult with the pastor of a local Russian church who is highly regarded in his community, and share with him the stories of some of the students who have been referred to you. As he listens, the pastor tells you that many families in his church had difficulty leaving Russia and, though they now live in the US, want to limit their interactions with American's as much as possible. Many parents wish to "preserve the Russian language and culture" and want to limit their children's exposure to English and American culture in general. Which of the following can you NOT conclude about the Russian students referred to you based on your conversation with the pastor? a. it is possible that instead of having language impairments, these students are typical language learners who have had limited assimilation into American mainstream society b. The families of these students who have been referred to you are bicultural, and this is part of the problem that the students are having in school c. the families of these students have limited acculturation into American life, and this factor may be influencing the student's performance in school d. The reluctance of the families to assimilate into American culture might be a factor that is influencing the student's interaction with other students who are native speakers of English, and this may be related to the Russian student's learning English more slowly

b. The families of these students who have been referred to you are bicultural, and this is part of the problem that the students are having in school

Studying the speech of a group of children who have developmental apraxia of speech, the researcher finds that the faster the children speak, the less intelligible they are. The researcher obtains a Pearson r correlational relationship of -.92. One could say that this shows that there is ____ between rate of speech and intelligibility. a. a positive correlational relationship b. a strong negative correlational relationship c. a canonical correlational relationship d. a cause-effect relationship

b. a strong negative correlational relationship

Health Insurance Portability and Accountability Act (HIPAA) includes, among other mandates a. a requirement that each state develop standards for protecting the privacy of medical patients to be reviewed and approved by the federal government b. a uniform federal standard for protecting the privacy of patients, to be followed by all medical, nonmedical, and allied health personnel in the country c. a stipulation that different health care entities involved in providing services to a patient cannot share information about that patient d. a requirement that when a request is received, health care providers have 90 days to provide access to the patients medical and related records

b. a uniform federal standard for protecting the privacy of patients, to be followed by all medical, nonmedical, and allied health personnel in the country

A father comes to you regarding his daughter, who is 8 months old. The daughter's hearing loss is bilateral, and she is profoundly deaf. The father states that he wishes for his daughter, as she grows older, to "fit in with children with normal hearing." He is interested in any possible amplification and says that he wants his daughter to lead a life that is "as normal as possible." Which training approach would best fit this father's wishes? a. total communication b. aural/oral method c. manual approach d. rochester method

b. aural/oral method

Purposeless, random, involuntary movements of body parts associated with hyperkinetic dysarthria is known as a. myoclonus b. chorea c. tics d. tremors

b. chorea

A new test of morphological skills, the Morphological Estimate of Student's Skills (MESS), correlates very well with a test of morphological skills that has been used nationally for the last 15 years. One could say that the MESS has good a. content validity b. concurrent validity c. construct validity d. predictive or criterion validity

b. concurrent validity

Primary progressive aphasia is a form of a. fluent aphasia b. dementia c. nonfluent aphasia d. subcortical aphasia

b. dementia

A clinician working with 3 year old girl, Josie, who has a severe speech sound disorder, found the child very uncooperative. She hid under the table, threw therapy materials, and cried. Ignoring the behaviors and admonishing her to sit quietly did not work. The clinician then began to praise Josie for sitting quietly for one minute and naming pictures; she also gave Josie a sticker for being a "good girl" during the preceding one minute. In gradual steps, the clinician extended the duration between verbal praise and presentation of stickers and eventually praised Josie only occasionally. The clinician has just used a procedure known as.. a. corrective feedback b. differential reinforcement of incompatible behavior c. time-out d. extinction

b. differential reinforcement of incompatible behavior

ESSA stands for a. elementary student success act b. every student succeeds act c. english speaking student act d. english studies success act

b. every student succeeds act

A type of research involving the effect of independent variables that have occurred in the past and the investigator is making a retrospective search for causes of events is called a. single-subject design research b. ex post facto research c. survey research d. ethnographic research

b. ex post facto research

a 90 year old woman in a skilled nursing facility is in the end stages of Alzheimer's dementia. The top treatment priority would be a. improving her sentence structure b. facilitating communication with the staff during daily routines c. working on her word retrieval skills d. increasing orientation to date and time

b. facilitating communication with the staff during daily routines

Amyotrophic lateral sclerosis results in which kind of mixed dysarthria? a. spastic-ataxic dysarthria b. flaccid-spastic dysarthria c. hypokinetic-spastic dysarthria d. flaccid-ataxic dysarthria

b. flaccid-spastic dysarthria

Dementia associated with Pick's disease is part of a. Alzheimer's disease b. frontotemporal dementia c. semantic varient of primary progressive aphasia d. the nonfluent varient of the primary progressive aphasia

b. frontotemporal dementia

Stuttering in preschool children is more likely on a. content words b. function words c. final words in a sentence d. vowels

b. function words

Reliability means that a test or measure a. measures what it purports to measure b. has scores that are consistent across repeated testing or measurement c. has items that adequately sample the full range of the skill being tested d. correlates well with an established test of known validity

b. has scores that are consistent across repeated testing or measurement

Which one of the following is false regarding treatment of children with SSD's? a. in maximal contrast therapy, also known as the maximal opposition approach, the selected word pairs contain maximum numbers of phonemic contrasts b. hodson and paden's cycles approach involves treating children with phonological disorders in cycles in which the child is trained to a criterion of mastery for error patterns such as final consonant deletion and fronting c. Van riper's approach focuses on phonetic placement, auditory discrimination/perceptual training, and drill like repetition and practice at increasingly complex motor levels until target phonemes are produced correctly in spontaneous conversation d. in minimal pair contrast therapy, the clinician uses pairs of words that differ by only one feature; of the paired words, one is the target word in which the sound is produced correctly and the other is the child's incorrect production

b. hodson and paden's cycles approach involves treating children with phonological disorders in cycles in which the child is trained to a criterion of mastery for error patterns such as final consonant deletion and fronting

You are evaluating a girl who has been referred because difficulties associated with partial submucous cleft palate accompanied by a bifid uvula. During your evaluation, you can probably expect to find a. hypernasality, leading to difficulty producing nasals adequately b. hypernasality, accompanied by decreased intraoral breath pressure, leading to difficulties with adequate production of fricatives, affricates, and plosives c. hyponasality, accompanied by increased intraoral breath pressure, leading to difficulties with adequate production of liquids and glides d. hypernasality, accompanied by difficulty producing vowels and nasals adequately

b. hypernasality, accompanied by decreased intraoral breath pressure, leading to difficulties with adequate production of fricatives, affricates, and plosives

Conduction aphasia is caused by lesions a. in the areas supplied by the middle cerebral arteries and the anterior and posterior arteries b. in the region between broca's area and wernicke's area, especially in the supramarginal gyrus and the arcuate fasciculus c. in brodmann's areas 44 and 45 in the posterior-inferior gyrus of the left hemisphere d. in the angular gyrus, the second temporal gyrus, and the juncture of the temporoparietal lobe

b. in the region between broca's area and wernicke's area, especially in the supramarginal gyrus and the arcuate fasciculus

You have been asked to counsel Doug, a 45 year old man who has smoked and drank alcohol since he was a teenager. He now has laryngeal cancer, and before surgery, the surgeon asks you to talk with Doug about esophageal speech. You explain to Doug that there are two basic types of esophageal speech. In one method, the patient is taught to keep the esophagus open and relaxed while inhaling rapidly. In the other method, the patient impounds the air in the oral cavity, pushes it back into the esophagus, and vibrates the cricopharyngeus muscle. The second method is called the a. inhalation method b. injection method c. inspiratory injection method d. laryngeal airway resistance method

b. injection method

As part of your job in a public-school district, you serve several preschools. You get an urgent call from a parent at one of the schools. She states that her daughter Michelle (age 4), who previously had typical skills, has suddenly "lost her words" and is having seizures. She shares that Michelle "is so much more hyper now, and we don't know what's wrong." The mom shares that they have an appointment with Michelle's pediatrician to find out more about what is happening. You support this decision, especially because you suspect that Michelle has a. williams syndrome b. landau-kleffner syndrome c. treacher collins syndrome d. pierre robin syndrome

b. landau-kleffner syndrome

Which one of the following is a homophenous pair? a. sheep-beep b. man-ban c. pan-fan d. honey-money

b. man-ban

You are observing treatment session in which a clinician is treating a 65-year-old man who has aphasia. In the previous sessions, the client had been reinforced for correctly naming several pictures. In the current session, the clinician shows 10 untrained stimuli and asks, "what is this?" for each picture/stimulus item. However, the clinician does not reinforce correct responses. In this latter procedure, the clinician is a. fading the reinforcers but differentially applying indirect reinforcement b. measuring generalized procedures with a probe c. fading the reinforcers d. assessing whether the client has learned to name the treatment stimuli

b. measuring generalized procedures with a probe

Rohini is a 6 year old speaker of Telegu from India. Her family moved to the US 6 months ago because her mother is a doctor and wanted expanded opportunities in the US Rohini speaks primarily Telegu and almost no English. Rohini is one of three children. Her mother confides that Rohini stands out from her two other siblings in that she "just didn't catch on to English" as fast as they did. She also shares that Rohini was slow to develop her language milestones in Telegu, not speaking her first word in Telegu until she was 2 years old. You are a monolingual English speaking SLP and know that given the red flags from the mother's description, you need to do a thorough evaluation to ascertain whether or not Rohini has a language disorder or just a language difference that will resolve with time. However, you do not speak Telegu. What is the best choice in this situation? a. not test Rohini at all- wait and see if time will make a difference b. obtain the services of a trained interpreter and have him or her evaluate Rohini in Telegu c. administer standardized tests in English because, after all, Rohini needs to succeed in American schools d. Have Rohini's mother fill out a detailed case history form and use the results to guide your decision

b. obtain the services of a trained interpreter and have him or her evaluate Rohini in Telegu

A clinical researcher, who has been using Van Riper's fluent stuttering approach to treating stuttering in adults, realizes that its effectiveness is unknown. He plans to design a study to find out if the method is effective. To make a valid conclusion, the researcher must use a. the survey method in which he would seek the opinions of other clinicians who use the method b. one of the experimental designs c. the case study method d. the retrospective research method

b. one of the experimental designs

You have been asked to assess Rudy, a 65-year old man, for suspected neurogenic stuttering. In your assessment, you expect to observe a. absence of repetitions on medial and final sounds in words b. presence of dysfluencies in imitated speech c. presence of marked adaptation d. significant number of sever associated motor behaviors

b. presence of dysfluencies in imitated speech

A researcher wants to study the occurrence of stuttering in a given city. She wants to know how many adults and children in Middletown City have officially been diagnosed with stuttering. She does not necessarily want to give a predictive statement; rather, she just wants to know the number of individuals in Middletown who stutter. The researcher wants to study the a. incidence b. prevalence c. matched samples d. population statistics

b. prevalence

A teacher has referred Isaiah Brown to you for an evaluation. Isaiah is an african american kindergartener who is reportedly doing well in class academically. When you observe him on the playground with his peers, you see that he has many friends and does not appear to have problems interacting appropriately with other children. Isaiah's friends do not appear to have any difficulty understanding what he says. However, the teacher is concerned because, she says, "I think Isaiah pronounces some of his sounds wrong. I think he needs speech therapy." When you conduct a speech screening with Isaiah, you will remember: Which of the following is a typical variation of African American English and not a sign of a disorder? a. v/f substitution in word-final position b. production of [æks] instead of [æsk] c. w/r substitution in all word position d. initial consonant deletion

b. production of [æks] instead of [æsk]

An employee in a tech company was able to routinely excuse himself from making oral presentations to his team because of his stuttering; provided other factors also support it, what would be your likely diagnosis of this client? a. typical stuttering b. psychogenic stuttering c. malingered stuttering d. neurogenic stuttering

b. psychogenic stuttering

A clinician evaluates the speech of a 5 year old child with a phonological delay. The child is not intelligible to her kindergarten teacher or her peers and is placed in therapy to improve her intelligibility. Assuming that this child uses the phonological process of consonant cluster reduction, which of the following is the word you would most likely put on a word list used for treatment? a. bus b. stopped c. rich d. lassie

b. stopped

Lynne is a 6 year old child with dysarthria of speech secondary to cerebral palsy. You can expect that a. labiodental fricatives will be the easiest sounds for her to produce b. stops, glides, and nasals will be easier for her to produce than fricatives, affricates, and liquids c. there is a strong possibility that her intelligibility will be impacted by hoarseness d. she will not show voicing errors when she produces sounds

b. stops, glides, and nasals will be easier for her to produce than fricatives, affricates, and liquids

A condition in which the surface tissues of the soft or hard palate fuse but the underlying muscle or bone tissues do not is called a. fusion disorder b. submucous or occult cleft palate c. class III palatal cleft d. submucosal cleft class IV

b. submucous or occult cleft palate

You are working as a clinician in a private clinic. A father brings his son, Johnny, age 4 and a half years, for an evaluation. According to his father, Johnny is "hard to understand, and sometimes the kids at preschool make fun of him." The pediatrician has told Johnny's father that Johnny will "outgrow this speech problem on his own," but the father wants to make sure that this advice is correct. Johnny will be starting kindergarten in 6 months, when he turns 5 years of age, and his father wants to be sure that Johnny speaks as intelligibly as possible so that he will not be teased in elementary school. When you evaluate Johnny, you find that he has th/s, t/f, w/r, d/th, and j/l substitutions. You decide to place him in therapy. You would like to begin therapy by addressing the a. th/s substitution b. t/f substitution c. w/r substitution d. d/th substitution

b. t/f substitution

Such skills as airflow management, gentle phonatory onset, and reduced rate of speech are targets in: a. the fluent stuttering technique b. the fluency shaping technique c. counseling to reduce psychological conflicts d. direct stuttering reduction strategies

b. the fluency shaping technique

A 5-year-old, Crystal S, is brought to you for an evaluation of her speech. The family speaks only English in the home. According to Crystal's mother, Crystal "loves to talk but most people have trouble understanding her." As you play with crystal informally, you estimate that she is approximately 50-60% intelligible. You conduct an oral peripheral exam, which reveals that she does not have any anatomical or physiological anomalies that would explain why she is so unintelligible. You also conduct in-depth assessment in other areas to determine the nature of her unintelligibility and to determine therapy goals. You find that crystal uses a number of phonological patterns. One of those patterns is stopping. You know this when you hear her make such substitutions as a. bæ/bæt b. to/so c. rvz/rvsh d. pEt/pEst

b. to/so

Key parts of the auditory nervous system include cranial nerve VIII, which has two branches: the _____ branch and the _____ branch, which carries the electrical sound impulses from the cochlea to the brain a. retrocochlear, vestibular b. vestibular, auditory-acoustic c. auditory-acoustic, retrocochlear d. cochlear, auditory-acoustic

b. vestibular, auditory-acoustic

As a clinician in a medically based private practice, you receive a referral of 37-year-old Jan, who has been a physical education teacher for the last 15 years. Jan works part time as a telemarketer, and, according to her husband, she "is glued to her cell phone." Jan also sings in the church choir. She has been hoarse for several years and tells you during the case history that "I've ignored the way I sound—it's just me. I haven't felt like I've needed to change anything." However, Jan shares that lately she has been feeling a lot of pain, and the hoarseness is substantially worse. She says, "Sometimes when I talk, it's almost like there's a 'double voice.'" Jan tells you she is worried because her job depends upon being able to yell and raise her voice on the playground as she teaches physical education. She is worried that she might lose her job if she can no longer yell. You immediately refer Jan to an otolaryngologist for a thorough examination of her vocal folds. You then proceed to do your own perceptual and instrumental evaluation. You arrive at a number of findings, including the fact that Jan has increased laryngeal dysphonia, airway resistance, and a maximum phonation time of 7 seconds. You think that Jan is a probable candidate for phonosurgery, but you will wait for the otolaryngologist's diagnosis and recommendations. You need to view Jan's vocal folds. You know that the otolaryngoogist will do this also, but you are fortunate to have instrumentation available to you. You decide to use a procedure that uses a pulsing light to permit the optical illusion of slow-motion viewing of the vocal folds. This is called a. electroglottography b. videostoboscopy c. electromyography d. videofluoroscopy

b. videostoboscopy

Which of the following is a predictable production for speakers of Asian languages as a result of language interference (transfer)? a. "He be to bed going now" b. "I see cat the little" c. "Yesterday she cook a pot of soup" d. "We no not be drivin' over there"

c. "Yesterday she cook a pot of soup"

A minimum of how many hours of supervised clinical experience is needed for an individual to be an SLPA? a. 150 b. 250 c. 100 d. 200

c. 100

You have been asked to give an in-service to a group of students who wish to eventually specialize in service delivery to children with cleft palates and their families. The students want to know detailed information about in utero development of the hard and soft palate (among other things). You can accurately tell them that in utero, the hard palate fuses between the developmental ages of a. 1 and 2 weeks b. 5 and 6 weeks c. 8 and 9 weeks d. 10 and 12 weeks

c. 8 and 9 weeks

Amanda is a 7 year old with childhood apraxia of speech (CAS). She is frustrated in school because it is hard for her to be understood in class and on the playground, you estimate that she is approximately 60% intelligible. You have just finished graduate school, and Amanda is your first client with CAS. When you are listening to Amanda speak, you notice the presence of intermittent hyper nasality that is somewhat unpredictable. There is also nasal emission on some consonants. These phenomena occur because a. Amanda probably has a shortened velum secondary to CAS b. Amanda's velum is weak and you will need to conduct oral motor exercises to strengthen it c. Amanda has impaired motor programming of the velum resulting in incompetent velar movement d. Amanda may have a partial sub mucous cleft palate

c. Amanda has impaired motor programming of the velum resulting in incompetent velar movement

An infant with cleft palate will most likely have hearing problems because of a. aural atresia b. an incompletely formed cochlea c. ET dysfunction d. malformed ossicles

c. ET dysfunction

The position that stuttering indicates a social role conflict was taken by a. Van Riper b. Wischner c. Sheehan d. Bloodstein

c. Sheehan

Passed in 1935, the federal government's first major step toward involvement in a widespread program of living and medical assistance to the population at large is known as the a. Americans with disabilities act (ADA) b. health insurance portability and accountability act (HIPPA) c. Social Security act (SSA) d. Individuals with disabilities education act (IDEA)

c. Social Security act (SSA)

A hospital based clinician is conducting an experiment to assess the efficacy of the new, exciting Newton Aphasia Program (NAP) in increasing the word-retrieval skills of her people with aphasia. The experimental and control groups have been carefully matched on all variables. Halfway through the experiment, the clinician finds that many of the experimental participants attend church each Sunday and go to Sunday school after church. In sunday school, there is bible study and a great deal of discussion. None of the control participants attends church or Sunday school. At the end of the experiment, the clinician finds that the experimental group of participants who received the NAP have improved significantly in their word-retrieval skills compared to the control participants, who have been treated with more traditional word-retrieval therapy techniques. What can the researcher safely conclude from her study? a. The NAP was more successful in helping the experimental patients improve their word retrieval skills that were the traditional methods in helping patients in the control group improve their word retrieval skills b. The NAP was not successful in helping the patients in the experimental group improve their word-retrieval skills; the improvement was due to weekly church and sunday school attendance c. The experimental groups sunday school and church attendance on a weekly basis was a possible confounding variable in the study, making it impossible to firmly conclude that the NAP alone caused the difference in the performance of the experimental and control groups d. The NAP was only moderately successful in helping experimental group patients improve their word-retrieval skills

c. The experimental groups sunday school and church attendance on a weekly basis was a possible confounding variable in the study, making it impossible to firmly conclude that the NAP alone caused the difference in the performance of the experimental and control groups

You are on a cleft palate and craniofacial anomalies team that evaluates children with cleft palates. When a child has a resonance problem, the team decides whether the child will benefit from speech therapy alone or more intensive medical intervention is needed. As part of the assessment process, cephalometric analysis is used. In cephalometric analysis, the cephalometric assessment of velopharyngeal structures computer program analyses the ratio relationship between the length of the soft palate and the depth of the nasopharynx. Which one of the following statements best summarizes a key principle that guides treatment decisions? a. a ratio of less than 60 is usually found when the soft palate is too short and the nasopharynx is too shallow b. a ratio of less than 60 is usually found when the nasopharynx is too deep or the soft palate is too long c. a ratio of 60-80 generally indicates adequate tissue for velopharyngeal closure for speech d. if a ratio is higher than 80, this means the nasopharynx is too shallow or the velum is too long

c. a ratio of 60-80 generally indicates adequate tissue for velopharyngeal closure for speech

As a clinician in a medically based private practice, you receive a referral of 37-year-old Jan, who has been a physical education teacher for the last 15 years. Jan works part time as a telemarketer, and, according to her husband, she "is glued to her cell phone." Jan also sings in the church choir. She has been hoarse for several years and tells you during the case history that "I've ignored the way I sound—it's just me. I haven't felt like I've needed to change anything." However, Jan shares that lately she has been feeling a lot of pain, and the hoarseness is substantially worse. She says, "Sometimes when I talk, it's almost like there's a 'double voice.'" Jan tells you she is worried because her job depends upon being able to yell and raise her voice on the playground as she teaches physical education. She is worried that she might lose her job if she can no longer yell. You immediately refer Jan to an otolaryngologist for a thorough examination of her vocal folds. You then proceed to do your own perceptual and instrumental evaluation. You arrive at a number of findings, including the fact that Jan has increased laryngeal dysphonia, airway resistance, and a maximum phonation time of 7 seconds. You think that Jan is a probable candidate for phonosurgery, but you will wait for the otolaryngologist's diagnosis and recommendations. You receive a phone call and a report from the otolaryngologist that Jan will indeed need phonosyrgery for the presence of bilateral vocal fold polyps (the right polyp is larger than the left). The otolaryngologist wants to obtain quantitative measurements of Jan's voice before phonosurgery; he wants to use these baseline measures as a comparsion with measures taken after phonosurgery to evaluate whether the phonosurgery was sucessful. To obtain these quantitative measurements, you will probably use a. esophageal manometry b. indirect laryngoscopy c. a sound spectrograph d. a plethysmograph

c. a sound spectrograph

In most states, to work in the public schools, SLPs and AuDs are required to possess: a. a state license b. ASHA certification c. a state-issued credential from an agency such as the department of ed d. state license and ASHA certification

c. a state-issued credential from an agency such as the department of ed

A 51-year-old woman comes to an audiologist and states that she feels like she is losing her hearing in her left ear. The woman says that she generally feels healthy but has noticed that she now uses her right ear exclusively when talking on the telephone. She states, "sometimes the left side of my face tingles. It doesn't bother me much, but I do notice it sometimes." She also reports that she feels slight dizziness and has notices some mild balance problems. For example, when she is on an escalator, she has to "hang on more." She states that "sometimes my left ear rings." Her preliminary audiological results show that she normal ability to detect pure tones and that she has normal speech recognition in a quiet room. The most probable diagnosis of this woman's problems is... a. central auditory processing disorder b. meniere's disease c. acoustic neuroma d. otosclerosis

c. acoustic neuroma

A 5-year-old, Crystal S, is brought to you for an evaluation of her speech. The family speaks only English in the home. According to Crystal's mother, Crystal "loves to talk but most people have trouble understanding her." As you play with crystal informally, you estimate that she is approximately 50-60% intelligible. You conduct an oral peripheral exam, which reveals that she does not have any anatomical or physiological anomalies that would explain why she is so unintelligible. You also conduct in-depth assessment in other areas to determine the nature of her unintelligibility and to determine therapy goals. You are conducting therapy with a highly unintelligible 5-year-old and are targeting phonological awareness skills (among other things). Two target sounds in therapy are /k/ and /g/. You carry out activities that focus on helping the child attend to the initial sound in target words. For example, you have her play a "fishing" game where she catches "fish" with words that begin with /k/ and /g/. After she catches a fish, she puts it into the appropriate "pond" You are targeting which specific phonological awareness skills? a. rhyming b. syllable segmentation c. alliteration d. syllabication

c. alliteration

Mrs. Elizaga, a 70 year old Filipino woman, has had a stroke. Her first and dominant language is Tagalog. She speaks some English. You do not speak Tagalog but know that you need to evaluate her Tagalog skills. As a culturally competent clinician, you will a. evaluate her in English only; after all, she is in the US and needs to be evaluated in our mainstream language b. ask a family member to come and gather a language sample in Tagalog and help you analyze it and figure out goals based upon this sample c. ask the medical facility to provide you a certified, highly trained Tagalog interpreter to gather a case history and evaluate Mrs. Elizaga's Tagalog skills, following up with some English assessment as well d. have Mrs. Elizaga's family fill out a form indicating whether or not Tagalog or English is her dominant language and asking for their input on best assessment practices

c. ask the medical facility to provide you a certified, highly trained Tagalog interpreter to gather a case history and evaluate Mrs. Elizaga's Tagalog skills, following up with some English assessment as well

A clinician in a hospital setting is asked to evaluate a 64-year-old patient who appears to have dysarthria as a result of a TBI from falling and hitting the back lower portion of his head. A detailed motor speech evaluation shows that the patient presents with slurred speech, imprecise consonants, distorted vowels, slow rate of speech, and excess and even stress. The patient's motor movements are described by the nursing staff as "clumsy" and "uncoordinated." Based on the given information, the clinician would most likely classify the patient's dysarthria as a. hyperkinetic dysarthria b. hypokinetic dysarthria c. ataxic dysarthria d. flaccid dysarthria

c. ataxic dysarthria

A child comes to you for an eval. according to mom, she has a history of middle ear infections. the mom reports she is quite difficult to understand. she says guck/duck and koo/too. This phonological process is: a. fronting b. stridency deletion c. backing d. glottal replacement

c. backing

A clinician in a hospital setting is informed that insurance companies have begun to demand specific evidence that the clients with neurologically based disorders are taught functional communication skills. These skills are a. age and norm based skills that are appropriate for the client b. useful only for adult clients c. behaviors that promote communication in natural settings d. useful only for clients with language disorders

c. behaviors that promote communication in natural settings

You are a new clinician, considering using a new test with an EL student. You are asking questions like "what theory was used in the test's creation?" "Is any theory mentioned?" or "Is it appropriate for this particular student?" You are questioning the test's a. concurrent validity b. reliability c. construct validity d. theoretical reliability

c. construct validity

You have received a referral of Akhtar, a 4-year-old refugee child from a Dari-speaking family from Afghanistan. Akhtar has never been evaluated or received speech-language services. Before you meet Akhtar, you speak with the doctor, who explains that Akhtar has midline dysplasia, a high and narrow arched palate, a relatively large and fissured tongue that protrudes, hearing loss, and speech sound disorder. Akhtar also has generalized hypotonia and brachycephaly (small head). You can confidently conclude that Akhtar has a. crouzon syndrome b. cri du chat syndrome c. down syndrome d. hurler syndrome

c. down syndrome

The belief that vocally abusive behaviors in humans lead to vocal nodules and the resulting voice disorder is supported by a. experimental research b. prospective research c. ex post facto or retrospective research d. experimental group control group research

c. ex post facto or retrospective research

You are asked to see an 8 year old boy, Jason, for potential therapy because he is very hoarse. Jason has been hoarse for approximately 8 months. He is an active, happy, 3rd grader who loves sports and is engaged in various types of sports (soccer, baseball, etc.) year-round. Reportedly, Jason frequently screams at games. At school, there is a 15 minute recess in the morning and 30 minute after lunch. You observe Jason on the playground at recess several times over a period of 2 weeks and see that he loves to run, play, and yell loudly with his friends. Jason's parents have given you a letter from the ENT that definitively states that Jason has vocal nodules. After an evaluation, the first thing you would do is a. prescribe 2-3 weeks of almost total voice rest, telling Jason and his parents that he can speak only when he absolutely has to- no yelling at recess or when he plays sports b. monitor Jason's vocal status by seeing him once every 2 months for the next year to observe whether his hoarseness gets better or worse c. focus on identification and reduction of vocally abusive behavior, such as yelling and screaming, using computer games to help motivate Jason to use better vocal habits d. give Jason and his parents reading materials that discuss vocal abuse and tell the parents that if the hoarseness does not resolve in 1-2 months, you will probably see Jason for voice therapy

c. focus on identification and reduction of vocally abusive behavior, such as yelling and screaming, using computer games to help motivate Jason to use better vocal habits

Of the following symptoms, the one associated with dysarthria is a. even and consistent breakdowns in articulation b. impaired syntactic structures c. forced inspirations and expirations that interrupt speech d. an invariably slower rate of speech

c. forced inspirations and expirations that interrupt speech

Children with bilateral cleft lip or palate may have problems with their teeth. A common problem for these children is a. supernumerary teeth b. teeth erupting out of the hard palate c. hypodontia or missing teeth d. teeth that are too small

c. hypodontia or missing teeth

Which of the following dysarthria is associated with parkinson's disease? a. spastic dysarthria b. flaccid dysarthria c. hypokinetic dysarthria d. hyperkinetic dysarthria

c. hypokinetic dysarthria

Which of the following would be inappropriate to recommend for a patient with dysphagia due to poor base-of-tongue retraction? a. use of the chin tuck strategy b. swallowing multiple times for one bolus c. lingual sweep of lateral sulci d. effortful swallow

c. lingual sweep of lateral sulci

You are evaluating Ronnie, a 3 year old boy who is moderately unintelligible. His phonetic inventory includes the phonemes /t, d, m, n, p, b/. He manifests the phonological pattern of final consonant deletion. In therapy, the most appropriate target word to focus on would be a. path b. horse c. map d. whistle

c. map

A hospital based clinician receives a referral of a woman, Fran, who is 76 years old and enjoys walking, swimming, and giving her grandchildren rides. During the initial interview, Fran tells the clinician that she would have to stop her daily walks with her dog, because she believes she is slow when she begins walking and then she would take short, rapid, shuffling steps. She also shares that her writing has become smaller and that her friends and family say that she has been found "expressionless" in recent days. The clinician also notices decreased intelligibility. other symptoms the clinician might expect Fran to manifest would include a. chorea, emotional outbursts, schizophrenic like behaviors, dysarthria b. hallucinations, mask-like face, confabulation c. mask-like face, slow voluntary movements, tremors in resting muscles, and disturbed posture d. circumlocutions, repetitive verbal responses, and festinating speech

c. mask-like face, slow voluntary movements, tremors in resting muscles, and disturbed posture

A clinician in private practice often administers the Word Abilities Keystone Evaluation-Upper portion test to the children with language disorders. One day, she becomes curious about the "typical score" of the children to whom she administers the WAKE-UP. The clinician takes the scores of 17 children and lines them up in order from lowers to highest. The clinician identifies a number below and above which the same number of scores existed. That number is called a a. mean b. mode c. median d. standard deviation

c. median

The difference between modeling and imitation is that a. imitation is a treatment procedure, and modeling is a treatment target b. imitation is usually superior to modeling as a treatment strategy c. modeling is clinician's behavior and imitation is client's behavior d. modeling is necessary at all stages of treatment, but imitation is necessary only in the initial stage of treatment

c. modeling is clinician's behavior and imitation is client's behavior

Which one of the following is a fact about stuttering adaptation? a. the greatest reduction in stuttering occurs only on the seventh reading b. there is transfer from one reading passage to the other c. most of the reduction in stuttering occurs by the fifth reading d. a higher magnitude of adaptation occurs with an increased time interval between readings

c. most of the reduction in stuttering occurs by the fifth reading

Dementia of the Alzheimer's type is caused by a. thiamine deficiency b. pathological changes of the corpus callosum c. neurofibrillary tangles and neuritic plaques d. deterioration of neurons in the brainstem

c. neurofibrillary tangles and neuritic plaques

A group of clinicians wishes to conduct research in a hospital setting. These clinicians work with clients who have voice disorders. Many of the clients are hoarse because they work in noisy factories where they shout a great deal during the work week. The clinicians devise a rating scale to evaluate the hoarseness of these clients during the evaluation sessions. The scale ranges from almost no hoarseness (1) to great amount of hoarseness (4). this type of scale would be called a: a. ratio scale b. nominal scale c. ordinal scale d. logarithmic scale

c. ordinal scale

A clinician in a private practice is approached by the parents of Tommy D., a 5 year old boy. The parents want to place Tommy in kindergarten in the fall but say, "we know there's something wrong with him- we're just not sure what." According to Tommy's parents, he is a "sweet, lovable boy who will go to anybody. He likes to sing a lot, too." Because the parents live in a rural area, health care access has been limited. After seeing Tommy for the first time, the clinician refers his parents to a neurologist because she suspects that Tommy has a syndrome. He is small for his age and has an elfin-like appearance characterized by a small chin, turned up nose, puffiness around the eyes, a long upper lip, and a wide mouth. His teeth are small and widely spaced. The clinician knows that she will probably end up seeing Tommy for intervention if his parents are able to bring him on a weekly basis. She will probably be working on which of the following goals? a. pragmatics, to increase Tommy's ability to interact with others b. oral-motor coordination, because children with this syndrome usually have oral-motor coordination problems, which contribute to decrease intelligibility c. overall expressive and receptive language, because children with this syndrome generally have IQs of 50-70 (although some have good language skills) d. morphological skills, because although children with this syndrome usually have above average IQs, they frequently delete bound morpheme from the beginnings and ends of words

c. overall expressive and receptive language, because children with this syndrome generally have IQs of 50-70 (although some have good language skills)

According to ASHA standards, which one of the following activities would not be within the scope of practice of a SLP? a. assessing and treating swallowing and upper aerodigestive disorders (infant feeding) b. developing and implementing augmentative and alternative forms of communication and prescribing AAC devices and systems c. prescribing hearing aids for children and adults with hearing loss d. screening hearing and offering auditory training and speech language services to the deaf and hard of hearing

c. prescribing hearing aids for children and adults with hearing loss

The therapy technique of phonetic placement is used to teach or establish a. auditory discrimination b. stimulability c. production of a phoneme in isolation d. minimal pair contrasts

c. production of a phoneme in isolation

A 5-year-old child, Marcus, has been identified as needing treatment for his stuttering. His parents report that he has been stuttering since he was 3 years old, and the stuttering has become worse. Now children tease him, and his parents are concerned that, when he enters kindergarten, the teasing will become worse. The clinician decides to use one of the direct stuttering reduction methods with Marcus. Select the appropriate technique. a. delayed auditory feedback and slowed speech b. auditory masking c. response cueing d. airflow management and parental counseling

c. response cueing

A 54 year old pastor, Rev. Johnson, has a stroke and takes a leave of absence from his job while he recovers. After a 3-4 month period, he goes back to work, which include preaching sermons on Sunday mornings and visiting church members who are sick. He says that he is 100% back, however, his parishioners and family notice that he tends to bump into people who walk on his left; when he writes, he does not use the left side of the page. He does not recognize parishioners whom he has known for 30 or more years until they begin to speak. When people tell jokes, he does not laugh or appear to understand the jokes as he used to. He does not detect sarcasm in other people's speech. Rev. Johnson probably has a. dementia b. wernicke-korsakoff syndrome c. right-hemisphere syndrome d. apraxia of speech

c. right-hemisphere syndrome

[Audiogram. R/L AC/BC 30/40 @ 250 and down to 65-70 @ 8000] You can assume that this patient has a. mixed hearing loss b. conductive hearing loss c. sensorineural hearing loss d. noise-induced hearing loss

c. sensorineural hearing loss

An experimental design involving only a few participants and focusing on individual performance would be called a a. case study design b. ex post facto design c. single-subject design d. single correlational design

c. single-subject design

If a test is being to see whether responses to the items on the first half of the test correlate with responses to the items on the second half, then that test is being evaluated for a. test-retest reliability b. interjudge reliability c. split-half reliability d. parallel form reliability

c. split-half reliability

The leading edge hypothesis a. suggests that speech disruptions are most likely on the most complex language structures b. discounts the concept of inherent vulnerability to speech problems c. suggests advanced structures about to be learned to induce speech disruptions d. is a new model of stuttering treatment

c. suggests advanced structures about to be learned to induce speech disruptions

In infants and children with cleft palates, Eustachian tube dysfunction is probably mostly related to the lack of contraction of the a. levator veli palatini muscle b. veli palatini muscle c. tensor veli palatini muscle d. palatopharyngeus muscle

c. tensor veli palatini muscle

Spondee words are a. poly syllabic words with at least two voiceless sounds b. two-syllable words with primary stress on the first syllable c. two-syllable words with equal stress on each syllable d. one-syllable words that are phonetically balanced

c. two-syllable words with equal stress on each syllable

In your private practice, a father brings his 5 year old son, Jordan, to see you. The kindergarten teacher has noticed that Jordan "talks kind of funny" and sometimes other children make fun of him. Justin was born with a cleft palate and has had repair surgery, and he has a history of conductive hearing loss secondary to otitis media with effusion. During your evaluation of Jordan's speech, you note that he is especially having difficulty producing affricates, fricatives, and plosives. This is probably because he is having continued difficulty with a. vocal nodules b. hypertyophied adenoids c. verlopharyngeal closure d. maxillary deficiency

c. verlopharyngeal closure

The fluent stuttering treatment a. aims at eliminating stuttering b. seeks normally fluent speech c. was developed by Van Riper d. was developed by Johnson

c. was developed by Van Riper

A clinician in a hospital setting is asked to evaluate a 64-year-old patient who appears to have dementia. In gathering the case history from the patients adult daughter, the clinician finds out that the patient began drinking alcohol as a 15-year-old and has been a heavy drinker since that time. A detailed evaluation shows that the patient presents with memory problems, difficulty processing abstract information, and visual-spatial deficits. This patient most likely has... a. dementia of the alzheimers type b. parkinson's disease c. wernicke-korakoff syndrome d. aphasia

c. wernicke-korakoff syndrome

The structure that regulates the body posture, equilibrium, and coordinated fine motor movements is the

cerebellum

When two or more sounds of differing frequencies are combined, the result is a

complex tone whose vibrations may be periodic or aperiodic

The anterior cerebral artery supplies blood to the a. corpus callosum and basal ganglia b. corpus striatum c. caudate nucleus and globus pallidus d. lateral surface of the cortex

corpus callosum and basal ganglia

In a single subject design, the following are true: I. The A phase is the no-treatment or baseline phase II. The B phase is the treatment phase III. The A phase is the treatment phase IV. The B phase is the no-treatment or baseline phase V. There is no control group a. III b. III, IV, V c. I, II d. I, II, V

d. I, II, V

P.L. 99-457 established mandated development of individualized family service plans (IFSPs) . Which one of the following is not required to be included in a child's IFSP? a. the child's present level of development b. the family's needs and strengths relating to the child's development c. the major goals for the child and family and services to be provided d. a review of the plan at 12 month intervals or more frequently if needed

d. a review of the plan at 12 month intervals or more frequently if needed

A person with otosclerosis often as an audiogram reflecting Carhart's notch is a. a specific type of sensorineural hearing loss characterized by a dip at 1000Hz b. a specific loss at 4000Hz, as indicated by both air and bone conduction testing c. specific losses at both 2000 and 4000Hz, as indicated by bone conduction testing d. a specific loss at 2000Hz, as indicated by bone conduction testing

d. a specific loss at 2000Hz, as indicated by bone conduction testing

Dysarthria is: a. a speech disorder in the absence of muscle weakness or paralysis b. a speech disorder never associated with aphasia c. a single disorder with a unitary etiology d. a speech disorder associated with muscle weakness or paralysis

d. a speech disorder associated with muscle weakness or paralysis

A client comes to a clinician seeking voice therapy. Chris is a 25 year old male to female transgender client who has undergone several procedures to become more feminine. She tells you that she is also taking estrogen. She shares that she needs help to speak in a more feminine way, but she does not know how to go about this. She is also dealing with emotional issues surrounding her gender reassignment. In this case, the clinician should ideally a. tell Chris that various surgical procedures, such as thyroplasty, are available and that having surgical procedures will be sufficient to help her change her voice to sound more feminine b. advise Chris that surgical procedures are unnecessary, but that voice therapy will help her to sound more feminine by teaching her new communication patterns, such as higher pitch and more feminine intonation patterns c. advise Chris that a combination of voice therapy and counseling will be the best way for her to sound more feminine and also receive emotional support as she deals with gender reassignment issues d. advise Chris that a combination of counseling, surgery, and voice therapy to teach her more feminine pitch levels and communication patterns would best serve her needs

d. advise Chris that a combination of counseling, surgery, and voice therapy to teach her more feminine pitch levels and communication patterns would best serve her needs

Research on the prevalence of stuttering has shown that a. familial incidence is higher than the general population b. sons of stuttering mothers run a greater risk than sons of stuttering fathers c. blood relatives of a stuttering woman run a greater risk of stuttering themselves than those of a stuttering man d. all of the above

d. all of the above

A 32-year old man, Frank, wants to go to law school. He is very bright, but stutters and has been working in minimum-wage jobs where he does not have to do much talking. Frank shares that he has passed the entrance examination to get into law school, but he is afraid to enroll in classes. He feels frustrated by his dilemma and says that he is experiencing a great deal of anxiety about his situation. The clinician decides to use the fluent stuttering method. This would improve a. not discussing Frank's feelings and attitudes but rather teaching and establishing skills such as airflow management, reduced rate, and easy onset of phonation b. encouraging Frank to discuss his feelings and attitudes about his stuttering, desensitizing him to his stuttering, and using procedures such as time-out and response cost c. using DAF, masking, and time-out, and response cost d. allowing Frank to discuss his feelings and attitudes toward his situation, desensitizing Frank to his stuttering and helping him to modify his stuttering through the use of such techniques as cancellations and pull-outs

d. allowing Frank to discuss his feelings and attitudes toward his situation, desensitizing Frank to his stuttering and helping him to modify his stuttering through the use of such techniques as cancellations and pull-outs

A clinician in a skilled nursing facility receives a note that Dick, a new 80 year old patient, has been transferred to her facility. The note states that Dick was assessed by the clinician in the previous SNF, but there is no diagnosis in the papers that have been sent from the previous clinician. However, the previous clinician reported that Dick manifested the following symptoms: general awareness of his speech problems, significant articulation problems, problems with volitional speech with relatively intact automatic speech, more difficulty with consonants than vowels, intonation and fluency problems, and trial-and-error groping and struggling associated with speech attempts. Therapy was recommended. Dick most likely has a. hyperkinetic dysarthria b. unilateral upper motor neuron dysarthria c. right hemisphere syndrome d. apraxia of speech

d. apraxia of speech

You are working with Mr. Thomas, who has been diagnosed with Parkinson's disease. Primary symptoms you can expect to see include a. difficulty with sequencing motor movements of speech b. word retrieval problems and agrammatism c. confusion, disorientation, and emotional outbursts d. bradykinesia, festination, and cogwheel rigidity

d. bradykinesia, festination, and cogwheel rigidity

Single subject design strategy a. allows extension of the results to others in the population as long as the study was done properly b. does not allow extension from a single study c. can never allow extension of results d. can allow extension through replications

d. can allow extension through replications

You are serving a preschool attended by 50 children. Most of them are 4 years old, and their parents are highly involved because they are trying to prepare their children for kindergarten. A father is concerned about Justin, his 4 and a half year old son. Justin is highly intelligible, but his father reports that he has difficulty with consonant clusters in words like spring, street, and squirrel. You tell the father that at 4 and a half, it is natural for Justin to have difficulties with words with consonant clusters. However, according to developmental norms, he should have little difficulty with which of the following consonant clusters because many 4 year olds have mastered them? a. clusters gl, fl, pl b. clusters pr, br, tr c. clusters kst, lk, nd d. clusters sp, st, sk

d. clusters sp, st, sk

A mother brings her 3 year old, Sasha, for hearing screening. She shares that Sasha has had many middle ear infections, which have been treated with antibiotics. The middle ear infections started when Sasha was 9 months old. The mother has been told by a friend that "middle ear tubes" might be good option for Sasha, but the mother states that she is afraid of the surgery that this would entail. After all, Sasha is only 3 years old, and her mother does not want her to be traumatized. The mother tells you, "Even though Sasha ignores me sometimes, I think everything will be okay. I just wanted to get checked out to be sure." The audiologist assesses Sasha, with the resulting audiogram shown below. What type of hearing loss does Sasha have, based on this audiogram. [BC at 0-10, AC at 35-50 both ears] a. mixed loss b. retrocochlear loss c. sensorineural loss d. conductive loss

d. conductive loss

A high school teacher refers a japanese speaking 10th grader, kosuke, to you for an evaluation. Kosuke and his family have been in the united states for 2 years; the father was brought to the united states for a 3 year computer project in a major American city. The family will be returning to japan next year. Kosuke's teacher says that, although he is doing well academically, he is "hard to understand sometimes." She says that the other students don't make fun of Kosuke, but they don't always seem to follow what he is saying. The teacher wonders if he could benefit from intervention for an "articulation disorder." You conduct a screening. Which one of the following articulatory-phonological characteristics would NOT be predictable based on Kosuke's first language of japanese? a. substitutions of a/æ (sock/sack, fong/fang) b. final consonant deletion (be-/bed) c. r/l confusion (laise/raise, clown/crown) d. d/t substitution (din/tin)

d. d/t substitution (din/tin)

The suprahyoid laryngeal muscles lie above the hyoid bone; they are sometimes called elevators. The suprahyoid muscles are the a. digastrics, geniohyoids, thyrohyoids, stylohyoids, genioglossus, and sternothyroids b. thyrohyoids, digastrics, stylohyoids, and hyoglossus c. geniohyoids, mylohyoids, stylohyoids, and genioglossus d. digastrics, geniohyoids, mylohyoids, stylohyoids, genioglossus, and hyoglossus

d. digastrics, geniohyoids, mylohyoids, stylohyoids, genioglossus, and hyoglossus

A mother calls a clinician and shares concerns about her child's speech. According to the mother, her daughter, Rachel, is difficult to understand. The mother describes Rachel's speech as "sort of rushed, and she kind of stutters sometimes." The clinician who tests Rachel concludes that Rachel clutters. Based upon this diagnosis, one would expect to see that Rachel a. has a rapid rate of speech but is intelligible and is probably secretly anxious about her speech b. is dysfluent but has clear articulation and no spoonerisms c. has excellent language skills and is highly dysfluent with no speech rate problems d. has a lack of anxiety or concern about her speech; uses spoonerisms; and has rapid, disordered articulation resulting in unintelligible speech

d. has a lack of anxiety or concern about her speech; uses spoonerisms; and has rapid, disordered articulation resulting in unintelligible speech

A male child's risk for developing stuttering is the greatest when a. his brother stutters b. his father stutters c. his cousin stutters d. his mother stutters

d. his mother stutters

Though cleft palate is often caused by genetic factors, it can also be related to mechanical factors. Which one of these is NOT a mechanical factor related to cleft palate? a. intrauterine crowding b. twinning c. uterine tumor d. illegal drug use by the birth mother

d. illegal drug use by the birth mother

A child with traumatic brain injury would most likely manifest which of the following? a. a higher familial incidence b. echolalia and obsessive talking c. hypersensitivity to touch, insistence on routines, lack of interest in human voices d. impaired word retrieval and comprehension, and lack of attention and memory problems e. repetitive and stereotyped behaviors

d. impaired word retrieval and comprehension, and lack of attention and memory problems

Which of the following is false regarding dental deviations? a. skeletal malocclusion refers to deviations in the shape and dimensions of the mandible and maxilla b. dental malocclusion refers to deviation in the positioning of individual teeth c. in class I malocclusion, the arches themselves are generally aligned properly, however, some individual teeth are misaligned d. in class II malocclusion, the maxilla is receded and the mandible is protruded

d. in class II malocclusion, the maxilla is receded and the mandible is protruded

Negative reinforcement a. decreases the behaviors b. is the same as punishment c. does not involve aversive events d. increases the frequency of behaviors

d. increases the frequency of behaviors

You are serving in a juvenile detention facility and receive a referral of Max, a 16 year old with documented fragile x syndrome. Which of the following will you expect to find when you evaluate Max? a. attention deficits, a large tongue, sleep disturbances, high protruding forehead b. speech sound disorders, normal intelligence, expressive language delays, micrognathia c. gargoylism, tracheal stenosis, brachycephaly, hypertonia d. intellectual disability and delays in pragmatic, semantic, phonologic, and syntactic aspects of language, with syntax being especially affected

d. intellectual disability and delays in pragmatic, semantic, phonologic, and syntactic aspects of language, with syntax being especially affected

As a clinician in a medically based private practice, you receive a referral of 37-year-old Jan, who has been a physical education teacher for the last 15 years. Jan works part time as a telemarketer, and, according to her husband, she "is glued to her cell phone." Jan also sings in the church choir. She has been hoarse for several years and tells you during the case history that "I've ignored the way I sound—it's just me. I haven't felt like I've needed to change anything." However, Jan shares that lately she has been feeling a lot of pain, and the hoarseness is substantially worse. She says, "Sometimes when I talk, it's almost like there's a 'double voice.'" Jan tells you she is worried because her job depends upon being able to yell and raise her voice on the playground as she teaches physical education. She is worried that she might lose her job if she can no longer yell. You immediately refer Jan to an otolaryngologist for a thorough examination of her vocal folds. You then proceed to do your own perceptual and instrumental evaluation. You arrive at a number of findings, including the fact that Jan has increased laryngeal dysphonia, airway resistance, and a maximum phonation time of 7 seconds. You think that Jan is a probable candidate for phonosurgery, but you will wait for the otolaryngologist's diagnosis and recommendations. Measures of jitter and shimmer are becoming more common in use with voice patients because they can be useful in early detection of vocal pathology. Although you suspect that the otolaryngologist will find obvious vocal pathology, given Jan's history of prolonged hoarseness, you still want to obtain measures of jitter and shimmer because these can serve as an excellent baseline- especially if Jan has phonosurgery. When you take these measures, you might expect to see a. a small amount of shimmer and large amount of jitter b. a large amount of jitter with only a small or moderate amount of shimmer c. large amounts of both jitter and shimmer, with more than 1 dB of variation across vibratory cycles when jitter is measured d. large amounts of both jitter and shimmer, with more than 1 dB of variation across vibratory cycles when shimmer is measured

d. large amounts of both jitter and shimmer, with more than 1 dB of variation across vibratory cycles when shimmer is measured

Which one of the following is not true, according to the Individuals With Disabilities Education Improvement Act of 2004? a. Testing must be administered in a way that is not racially or culturally discriminatory b. testing and evaluation materials must be provided and administered in the language or other mode of communication in which the child is most proficient unless it is clearly not feasible to do so c. Testing must be administered to a bilingual child so as to reflect accurately the child's ability d. mandatory consent in the primary language is always required

d. mandatory consent in the primary language is always required

A hospital based clinician receives a referral of a woman, Fran, who is 76 years old and enjoys walking, swimming, and giving her grandchildren rides. During the initial interview, Fran tells the clinician that she would have to stop her daily walks with her dog, because she believes she is slow when she begins walking and then she would take short, rapid, shuffling steps. She also shares that her writing has become smaller and that her friends and family say that she has been found "expressionless" in recent days. The clinician also notices decreased intelligibility. based on your diagnosis (parkinsons), you would expect Fran's speech and language to be characterized by a. fluency problems, including silent pauses as well as repetitions because of false starts and attempts as self-correction b. quality and rate that are "drunken" and slow, with excessive and even stress c. incoherent, slurred, and rapid speech accompanied by metathetic errors d. monopitch, a harsh and breathy voice, short rushes of speech, imprecise consonants, and respiratory problems

d. monopitch, a harsh and breathy voice, short rushes of speech, imprecise consonants, and respiratory problems

Which of the following is characteristic of apraxia of speech but not dysarthria? a. consistent errors regardless of length and complexity of utterance b. difficulty performing both non-speech and speech motor tasks c. predictable errors d. normal strength, tone, and range of movement of oral and pharyngeal muscles

d. normal strength, tone, and range of movement of oral and pharyngeal muscles

The most significant communication problem associated with right hemisphere disorder is a. agrammatic speech b. impaired morphologic production c. severe voice disorders d. overall communication effectiveness

d. overall communication effectiveness

A hospital based clinician receives a referral of a woman, Fran, who is 76 years old and enjoys walking, swimming, and giving her grandchildren rides. During the initial interview, Fran tells the clinician that she would have to stop her daily walks with her dog, because she believes she is slow when she begins walking and then she would take short, rapid, shuffling steps. She also shares that her writing has become smaller and that her friends and family say that she has been found "expressionless" in recent days. The clinician also notices decreased intelligibility. Fran probably has a. right hemisphere syndrome b. unilateral upper motor neuron dysarthria c. alzheimer's dementia d. parkinson's disease

d. parkinson's disease

A hospital-based clinician receives a referral of Mary, a 71-year-old woman. Mary's 35-year-old son says, "Mom just isn't herself anymore; we don't know what's wrong or what to do. We don't know if she had a stroke or what's going on." After talking with Mary in the initial interview, the clinician realizes that he will have to assess Mary in depth to evaluate whether she has aphasia or DAT. If Mary has DAT, which of the following symptoms will she show? a. normal syntax except for word-finding problems; good auditory comprehension of spoken language; slurred and rapid speech; disorientation; visuospatial problems; difficulty with self-care and daily routines; intact repetition skills b. severe problems in recalling remote and recent events; relatively intact syntactic skills; appropriate humor; disorientation; intact ability to initiate interactions c. severely impaired fluency; severe echolalia; agrammatic and telegraphic speech; intact auditory comprehension skills; no difficulty with self-care or managing daily routines d. poor judgement; impaired reasoning; disorientation; widespread intellectual deterioration; empty speech; jargon; incoherent; slurred; rapid speech; problems in comprehending messages

d. poor judgement; impaired reasoning; disorientation; widespread intellectual deterioration; empty speech; jargon; incoherent; slurred; rapid speech; problems in comprehending messages

Which of the following is not a disorder of the pharyngeal phase of the swallow? a. decreased laryngeal elevation b. reduced tongue base retraction c. decreased upper esophageal sphincter opening d. premature spillage over the base of the tongue

d. premature spillage over the base of the tongue

A child is referred to you by his preschool teacher. This child, Damian, is 4 years and 3 months old and has transferred from out of state. In his previous state, he was reportedly assessed by an SLP who recommended that he receive intervention before kindergarten. According to the report from the previous clinician, Damian uses the phonological patterns of gliding, consonant cluster reduction, stopping, reduplication, and final consonant deletion. Your assessment confirms the presence of these phonological patterns. You would begin treatment by addressing a. final consonant deletion b. gliding c. consonant cluster reduction d. reduplication

d. reduplication

Jasmine D is a fourth grade girl who is struggling academically. She has difficulty paying attention in class, and her grades are low. She has been tested by several specialists, and there is no evidence of a learning disability or clinically significant attention issue. She has passed routine school hearing screenings every year; these screenings have been conducted at 25 dB bilaterally. However, teachers over the years keep referring her because they think that she is not hearing as well as she should. What course of action would you take? a. Dont do anything, she has passed hearing screenings and probably just needs to exhibit greater self-discipline in class b. talk to her parents and suggest ways that they can help her improve her behavior c. talk to her teacher and suggest discipline strategies for the classroom to help jasmine behave and focus better d. refer jasmine and her parents to an audiologist for a comprehensive hearing test

d. refer jasmine and her parents to an audiologist for a comprehensive hearing test

A 67 year old man comes to you for a voice evaluation. He was referred by his primary care doctor. He states that his voice has been getting "weaker" for the past 5-6 months. Upon oral peripheral examination, you find that he has fasciculations (tremors) of the tongue and some general facial weakness. The first thing you would do is a. refer him to a psychologist for an evaluation b. take detailed notes and tell him to come back in 6 months c. begin voice therapy, focusing on strengthening exercises d. refers him to a neurologist for an evaluation

d. refers him to a neurologist for an evaluation

Too many treatment procedures to treat a single communication disorder is one of the perplexities clinicians face. To survive this perplexity and to select a treatment procedure for a given disorder, the clinician should a. find out the most well-established procedure b. read the survey research reports to find out the most popular technique c. consult the manuals of various treatment procedures d. select a technique with the highest level of evidence available

d. select a technique with the highest level of evidence available

Vivian, a 72 year old woman, has just had surgery for laryngeal cancer. The clinician is trying to support Vivian in many ways, including asking several laryngectomy patients from a local support group to come and talk with Vivian about her options for speech. The support group members strongly recommend the Blom-Singer prosthetic device. They explain that the device is used by laryngectomees to a. shunt air from the esophagus to the trachea so that the salpingopharyngeus muscle will vibrate during inhalation b. assist in the development of competent esophageal speech c. prevent particles of food from entering the trachea d. shunt the air from the trachea to the esophagus so that the patient can speak on pulmonary air entering the esophagus

d. shunt the air from the trachea to the esophagus so that the patient can speak on pulmonary air entering the esophagus

Patients who might be treated with CO2 laser surgery, recurrent laryngeal nerve resection, botox, voice therapy, or a combination would probably have: a. contact ulcers b. paradoxical vocal fold motion c. hemangioma d. spasmodic dysphonia

d. spasmodic dysphonia

A 5-year-old, Crystal S, is brought to you for an evaluation of her speech. The family speaks only English in the home. According to Crystal's mother, Crystal "loves to talk but most people have trouble understanding her." As you play with crystal informally, you estimate that she is approximately 50-60% intelligible. You conduct an oral peripheral exam, which reveals that she does not have any anatomical or physiological anomalies that would explain why she is so unintelligible. You also conduct in-depth assessment in other areas to determine the nature of her unintelligibility and to determine therapy goals. You discover through your assessment that there are some sounds that Crystal consistently misarticulates. For example, she usually makes a t/k substitution. You want to know if she can produce /k/ in isolation. You show her how to produce /k/ by giving her a model, and you tell her "watch me make the /k/ sound. Then you do it just like I did" When you are doing this, you are assessing Crystals a. phonological knowledge b. receptive phonology skills c. overall intelligibility d. stimulability

d. stimulability

A singer comes to you for therapy. she had bypass surgery, and in the process, there was damage to her recurrent laryngeal nerve. In the course of intervention, you will most likely focus on: a. blowing exercises for more precise direction of her airstream b. abdominal exercises to strengthen the foundation for respiration c. chewing exercises to improve overall oral coordination d. strategies to improve vocal fold adduction

d. strategies to improve vocal fold adduction

Amanda is a 7 year old with childhood apraxia of speech (CAS). She is frustrated in school because it is hard for her to be understood in class and on the playground, you estimate that she is approximately 60% intelligible. You have just finished graduate school, and Amanda is your first client with CAS. Amanda has come back from a beach vacation with her parents, and excitedly tells you that she built a "thand castle" on the beach. Amanda has just manifested which type of articulation errors a. addition b. distortion c. coalescence d. substitution

d. substitution

If this test has adequate construct validity then: a. several judges have agreed that the test has been constructed appropriately and measures what it purports to measure b. test items are relevant to measuring what the test purports to measure c. the test accurately predicts future performance on a related task d. test scores are consistent with theoretical concepts or constructs

d. test scores are consistent with theoretical concepts or constructs

A clinician in a private practice is approached by the parents of Tommy D., a 5 year old boy. The parents want to place Tommy in kindergarten in the fall but say, "we know there's something wrong with him- we're just not sure what." According to Tommy's parents, he is a "sweet, lovable boy who will go to anybody. He likes to sing a lot, too." Because the parents live in a rural area, health care access has been limited. After seeing Tommy for the first time, the clinician refers his parents to a neurologist because she suspects that Tommy has a syndrome. He is small for his age and has an elfin-like appearance characterized by a small chin, turned up nose, puffiness around the eyes, a long upper lip, and a wide mouth. His teeth are small and widely spaced. This syndrome is caused by a rare genetic disorder that affects an estimated 1 out of every 20,000 babies. It is caused by.. a. a missing part of chromosome 22, known as 22q11 b. an expanded number of cytosine-guanine nucleic acid repeats on a specific gene on one of the distal ends of the x chromosome c. a spontaneous autosomal dominant mutation, whose gene and locus is FGR2 at 10q25-26 d. the deletion of approximately 25 genes on one copy of chromosome 7q11.23

d. the deletion of approximately 25 genes on one copy of chromosome 7q11.23

P.L. 94-142, The education of the handicapped act was later reauthorized and retitled as a. the americans with disabilities act b. the education of disabled individuals act c. the handicapped individuals education act d. the individuals with disabilities education act

d. the individuals with disabilities education act

Speech reception thresholds (SRTs) are a. determined by the patients response to a list of monosyllabic words presented at a low level of hearing b. determined by looking at the patient's pure-tone test results at the frequencies most important to speech c. the lowest or softest level of hearing at which a person can understand 100% of the words presented d. the lowest or softest level of hearing at which a person can understand 50% of the words presented

d. the lowest or softest level of hearing at which a person can understand 50% of the words presented

Studies on the rates of natural recovery of stuttering suggests that a. less than 20% of children who stutter recover naturally b. all children who stutter recover by age 8 c. once started, stuttering persists in all children d. the recovery rate may be as low as 45% or as high as 90%

d. the recovery rate may be as low as 45% or as high as 90%

Which of the following is not true? a. many state licensure laws are modeled after ASHA's requirements b. violation of ASHA's code of ethic can have major consequences, including revocation of ASHA's clinical certificate and cancelation of ASHA membership c. A certificate of clinical competence and state license are not necessarily required to practice in public school settings d. to receive a CCC in SLP or AuD, a SLP or AuD needs only to have a state granted credential to work in public schools

d. to receive a CCC in SLP or AuD, a SLP or AuD needs only to have a state granted credential to work in public schools

You observe a Spanish speaking child on the school playground. The teacher is concerned about Manuela's language skills, and you are starting by general observation in regular school settings. You hear Maneula speaking with her friends and saying things like "Yulie (Julie) has a shirt red," and "I want to estart playing with duh tederball (tetherball)." You are observing evidence of a. a clinically significant speech sound disorder b. a clinically significant language disorder c. clinically significant speech and language disorders d. transfer from spanish

d. transfer from spanish

A classroom teacher refers a 9 year old african american male student to you because she is concerned about his intelligibility. This teacher is anxious to avoid the mistake of mislabeling this student as having a speech disorder if he is merely manifesting characteristics of AAE. When you screen the boy, you find that he makes the following substitutions: d/m, f/n, and m/n. You would... a. let the classroom teacher work with the student because this is a mild problem b. do nothing, knowing that boys mature slower than girls c. do nothing, realizing that this is normal for speakers of AAE d. treat the student, because this is a sign of speech sound disorder involving substitutions of other sounds for nasals

d. treat the student, because this is a sign of speech sound disorder involving substitutions of other sounds for nasals

A teacher refers Jose to you for a speech-language evaluation. Jose a Puerto Rican American second grader who speaks Spanish and English with equal fluency, transferred to your school district 3 months age from another district in your state. In his previous district, he was in a bilingual classroom where his primary language of Spanish was maintained and he was exposed to English, also. According to his report card from the previous district, Jose does well speaking both Spanish and english. The teacher thought he is beginning to show a preference for English. Jose is performing adequately in all academic areas. The second-grade teacher at your school, who teaches only in English, feels that after 3 months in her classroom, Jose is catching on slowly. She wonders if he needs special education. Your best course of action would be to... a. ask Jose's parents to sign a permission form so that he may be assessed immediately in English using only English tests, since English is apparently beginning to be his preferred language b. use a variety of english screening instruments to screen Jose's English ability because these instruments are ecologically valid for him c. do nothing at the present time and tell the teacher that you will wait for 6 months to see how Jose progresses in his classroom d. use a dynamic assessment model to evaluate Jose's language-learning ability and combine this with classroom observations over the next 2-3 months to evaluate his progress

d. use a dynamic assessment model to evaluate Jose's language-learning ability and combine this with classroom observations over the next 2-3 months to evaluate his progress

Respiration relies on the muscles of inspiration and expiration. The thick dome-shaped muscle that separates the abdomen from the thorax is called the:

diaphragm

An important structure adjacent to the brainstem that contains the hypothalamus (which control the emotions) and the thalamus (which relays sensory impulses to various portions of the cerebral cortex) is called the:

diencephalon

The neurons that transmit information away from the brain are called

efferent neurons

A baby, Jason, is looking at the family cat. His grandma sees him looking at the cat and directs her gaze toward the cat, also. She prepares to comment about the cat. Jason's grandma is...

following Jason's line of regard.

A 5-year-old child has been referred to you for a language assessment. There is a concern about his expressive language skills, and you decide to gather a language sample to assess expressive morphology and syntax. At one point, when looking at a book, the child points to a book character and says, "him no eat cookies." This is an example of

four words, five morphemes, personal pronoun + one negative + one verb + one plural noun

When a person is producing voiced and voiceless /th/ the muscle of that is most involved is the a. palatoglossus b. sternocleidomastoid c. genioglossus d. styloglossus

genioglossus

The corpus striatum is composed of three nuclear masses, which are the

globus pallidus, caudate nucleus, and putamen

In a periodic complex sound, tones that occurs over the fundamental frequency and can be characterized as whole-number multiples of the fundamental frequency are called

harmonics

A child has been referred to you for an assessment of his pragmatic skills. The chief complaint of adults and children with whom he interacts is that he frequently gives commands and sounds rude and bossy. His classroom teacher says she is "fed up with bossiness," and peers do not include him in their games. His father tells you that the boy frequently says things like "take me to pizza palace" and "get me the spiderman DVD." The father would like intervention to help his son say things like "I wonder if we could get a spiderman DVD at the store," instead of giving orders. In therapy, you know you will need to work on the boys facility with

indirect requests

Which muscles from the list below are the most involved in adducting the vocal folds? a. lateral cricoarytenoids and transverse arytenoids b. digastricus c. cricothyroids d. posterior cricoarytenoids

lateral cricoarytenoids and transverse arytenoids

The structure at the inferior portion of the tongue that connects the tongue with the mandible is called the

lingual frenum

The /r/ and /l/ sounds may both be categorized as

liquids

A young child who says "down" when a cup of juice spills off of the dinner table is using the relation of

locative action

You observe a clinician working with a child who has a language impairment. They are making cookies together and the clinician is saying things like "look, the dough goes in the bowl; the spoon is beside the bowl. We will set the bowl on top of the counter and then make the cookies. We'll put them in the oven, and take them out when they are done." The clinician is working on developing the child's skill in the area of understanding: a. indirect requests b. locatives c. pragmatics d. gerunds

locatives

Compared to unstressed syllables, stressed syllables are typically

longer and higher in pitch

The laryngopharynx and the oropharynx add resonance to sounds produced by the larynx. The nasopharynx adds noticeable resonance to which sounds?

m, n, ng

The two properties of a medium that affect sound transmission are

mass and elasticity

During the typical speech production, vowels

may stand alone

The primary muscle of the lips is the

orbicularis oris

The back-and-forth movement of air molecules because of a vibrating object is referred to as

osillation

Muscles that contribute to velopharyngeal closure through tensing or elevating the velum are the a. tensor veli palatini, levator veli palatini, salpingopharyngeus b. stylopharyngeus, salpingopharyngeus, levator veli palatini c. levator veli palatini, genioglossus, salpingopharyngeus d. palatoglossus, tensor veli palatini, levator veli palatini

palatoglossus, tensor veli palatini, levator veli palatini

A young child who often says things like "my doggy" or "her ball" is using the relation of

possession

Which branch of the vagus nerve (CN X) innervates the cricothyroid muscle? a. superior laryngeal nerve b. lateral laryngeal nerve c. recurrent laryngeal nerve d. pharyngeal branch

superior laryngeal nerve

You have been asked to assess the language skills of 6-year-old Jennifer, who has been referred by her classroom teacher. The teacher says that Jennifer "talks in these really short sentences. I don't know if she is just shy or if there is more going on." The teacher has worked on oral language skills daily with her class. The end of the year is coming soon, and the teacher is concerned about how Jennifer will perform in second grade. You decide to conduct an informal language screening to decide whether you need to formally evaluate Jennifer's expressive language skills. You find that she uses many sentences, such as "he has a ball" and "I like pokemon." She uses few compound or complex sentences. You talk with her parents and find that this performance is also typical at home. Your next step would be to...

tell the teacher and parents that you would like to formally evaluate Jennifer's language skills because at 6 years of age, she should have an average MLU of 6.0-8.0, and her language should approximate the adult model

A first grade refers 6-year-old Mandy to you for an assessment. The teacher is concerned, because reportedly Mandy has problems with remembering what she hears. The teacher tells you, "sometimes I have to give the children three or four directions, and I have to do it quickly because we have to go somewhere, like an assembly. Mandy is the only one in my class who doesn't remember what I tell the kids to do." Based on this brief description, you suspect that Mandy might have difficulties in which of the following areas? a. temporal auditory processing b. divergent semantic production c. phonological processing d. convergent semantic production

temporal auditory processing

A sinusoidal wave is a sound wave

that is a result of simple harmonic motion

The term coda refers to

the consonant at the end of the syllable

The manner of articulation refers to

the degree of type of constriction of the vocal tract during consonant production

The lowest frequency of a periodic wave is also known as:

the fundamental frequency or first harmonic

The term coarticulation refers to

the influence of one phoneme upon another in production and perception, wherein two different articulators move simultaneously to produce two different speech sounds

Broad phoneme transcription involves

the use of IPA symbols to transcribe phonemes by enclosing them within slash marks (e.g., /f/)

Which one of the following is NOT a goal of the common core state standards? a. to create globally competitive citizens in the 21st century b. to prepare students for college c. to ensure that all students speak at least two languages so they become more competent global citizens d. to help students become responsible citizens who use evidence for deliberation

to ensure that all students speak at least two languages so they become more competent global citizens

Diacritical marks are

useful in making narrow phonetic transcription that gives more detailed information on a speaker's phonetic characteristics

A natural frequency is a frequency

with which a source of sound vibrates naturally

A fourth-grade child, Alex, has been referred to you for language testing by his teacher. His parents are concerned and upset with the teacher because they feel that Alex needs more help in reading and writing skills than he is receiving. They tell you that the math and science homework assignments are too difficult for him, and they feel that the fourth-grade teacher is making unreasonable demands. You find out that Alex did not attend preschool, and even in kindergarten, the teacher wrote on his first-trimester progress report that he "began school not knowing basic concepts; he didn't talk as much as the other children either." You will tell Alex's parents that...

you would like to conduct an assessment of Alex's language skills in a variety of domains to see whether he needs support services in oral and written language

The "typical" speaker of Standard American English would produce the word emancipation as:

/imænsəpeIʃən/

If a speaker said, "I just love 'em and leave 'em," the phrase "leave 'em" could be transcribed as:

/liv m,/

A semivowel that can be categorized as a voiced bilabial glide that is +anterior and +continuant is the:

/w/

In order to begin producing two-word combinations, how many words does a toddler need to have in his expressive vocabulary?

50


Conjuntos de estudio relacionados

BA 324H -- Exam One (Guffey 1-6)

View Set

Human Survey and Disease integumentary System

View Set

BLAW 3201 LSU FRY Chapter 11 "Study Guide"

View Set

Prowse REST OF IT pt1 (chs. 3,. 5, 10, 11 and. 12)

View Set

Corps et âme , de l'esprit DS2 Janvier : corps et pensée, esprit, liberté ( Spinoza, LEIBNIZ, DESCARTES......)

View Set

(Español III) El Calendario Cristiano

View Set

Топографія нижнього поверху черевної порожнини

View Set